Unit 2: Obstetrics - B. Abnormal Obstetrics

Lakukan tugas rumah & ujian kamu dengan baik sekarang menggunakan Quizwiz!

A 20-year-old G1P1 woman delivered her first baby 24 hours ago. Delivery was uncomplicated and she had an epidural placed for analgesia at 5 cm of cervical dilation. Earlier in the afternoon, she was complaining of a headache and was given ibuprofen. Three hours later, she complained of increasing headache, photophobia and nausea. She denies heavy bleeding. Vital signs are pulse 110; respirations 20; temperature 101.5° F (38.6° C); and blood pressure 100/50. Physical examination reveals obvious distress, as she has her eyes covered and pain when she moves her neck. Her lungs are clear and heart has a regular rate. Her abdomen is nontender, and uterine fundus is easily palpable just below the umbilicus and is nontender. Her extremities reveal no erythema, swelling or tenderness. Which of the following would most helpful to establish a diagnosis in this patient? A. Chest x-ray B. Urinalysis C. Lumbar puncture D. CBC with differential E. Pelvic ultrasound

C. Likely meningitis, perhaps from the epidural.

A 32-year-old G1P1 woman delivered a 9-pound baby and sustained a 4th-degree laceration two days ago. The delivery was complicated by a shoulder dystocia. Her laceration was repaired in layers in the customary fashion. She now complains of increasing pain in her perineal area, fever, chills and weakness. Her vital signs are: blood pressure 90/50; pulse 120; and temperature 102.2° F (39° C). Her abdomen is soft, nontender and her uterine fundus is firm and nontender. Her perineum is erythematous, swollen, but the laceration edges have separated and are grey. The laceration site is nontender and without feeling but there is tenderness of the surrounding tissue. In addition to broad spectrum antibiotics, what is your next step in the management of this patient? A. Sitz baths B. Whirlpool therapy C. Debridement D. Repair of laceration site E. Incision and drainage of perineal laceration

C. Aggressive debridement of the necrotic areas is required to prevent further spread of the infection. Debridement should extend until vital tissue with good blood supply is encountered. Repair of the defect should be delayed until the infection has completely resolved. Sitz baths and whirlpool therapy will provide symptomatic relief for her discomfort, but not adequate treatment. Incision and drainage of perineal laceration is appropriate for an uncomplicated abscess.

A 25-year-old G2P1 woman states her gestational age by known LMP is 16 weeks, 3 days. She reports no complaints and is not yet feeling fetal movement. Her fundal height is 22 cm. The MSAFP (maternal serum alpha fetoprotein) result is elevated. Which of the following is the most likely cause for the abnormal MSAFP result? A. Fetal trisomy B. Polyhydramnios C. Twin gestation D. Fetal abdominal wall defect E. Fetal neural tube defects

C. Alpha fetoprotein (AFP) levels in twin gestations are elevated and should be roughly twice that seen in singleton pregnancies. An additional clue to a possible diagnosis of twin gestation is the fundal height exceeding gestational age in weeks. Other causes of elevated maternal serum AFP include neural tube defects, pilonidal cysts, cystic hygroma, sacrococcygeal teratoma, fetal abdominal wall defects, and fetal death. Polyhydramnios is not by itself associated with abnormal MSAFP levels.

A 24-year-old G2P1 woman at 42 weeks gestation presents in early labor. At amniotomy, there is thick meconium and variable decelerations are noted. An amnioinfusion is started. Which of the following is most likely to decrease in this patient? A. Admission to the neonatal intensive care unit B. Post maturity syndrome C. Repetitive variable decelerations D. Risk for Cesarean section E. Meconium presence below the vocal cords

C. Amnioinfusion is a procedure where normal saline is infused into the intrauterine cavity. Amnioinfusionremains a reasonable approach in the treatment of repetitive variable decelerations, regardless of amniotic fluid meconium status. Meconium staining of the amniotic fluid is three to four times more common in the postterm pregnancy. This is likely due to two reasons: 1) greater length of time in utero allows for activation of a more mature vagal system; and 2) fetal hypoxia. Routine prophylactic amnioinfusion for thick meconium does not appear to decrease the incidence of meconium aspiration syndrome or have an impact on neonatal outcomes. Based on current literature, routine prophylactic amnioinfusion for meconium-stained amniotic fluid is not recommended.

A 33-year-old G2P1 presents at 36 weeks gestation for consultation because ultrasound revealed a fetus with biometry consistent with 30 5/7 weeks gestation. The EFW is less than the 5th percentile. Umbilical artery Doppler studies are abnormal. There is reverse end diastolic flow and the amniotic fluid volume is decreased. The AFI is 1.1 cm. Which of the following is the most appropriate next step in the management of this patient? A. Close observation with twice weekly NSTs and amniotic fluid assessments B. Close observation with twice weekly NSTs, amniotic fluid assessments and weekly umbilical artery Doppler studies C. Induction of labor D. Induction of labor at term (37 weeks gestation) E. Delivery by Cesarean section

C. Delivery is indicated in a fetus with IUGR at 36 weeks gestation with oligohydramnios and abnormal umbilical artery Doppler studies. Although there is an increased incidence of fetal intolerance of labor, induction of labor is generally preferred over elective Cesarean delivery. Delivery at term is indicated in fetuses with IUGR with reassuring fetal testing including a normal amniotic fluid volume.

A 38-year-old G0 woman presents for a preconception evaluation. She has a history of long-time anxiety and depression, and is interested in continuing her medications in pregnancy, which includes sertraline (Zoloft). She expresses concern because sertraline is a Category C drug. Which of the following descriptions is associated with FDA Category C drug classification? A. Studies in humans have demonstrated fetal abnormalities and/or there is positive evidence of human fetal risk based on adverse reaction data B. Positive evidence of human fetal risk based on adverse reaction data from studies in humans C. Animal reproduction studies have shown an adverse effect on the fetus and there are no adequate and well controlled studies in humans D. There are no adequate and well controlled studies in pregnant women; animal reproduction studies have failed to demonstrate a risk in the fetus E. Adequate, well-controlled studies in pregnant women that have not shown an increased risk of fetal abnormalities to the fetus in any trimester of pregnancy

C. Depression is more common in women than men. Appropriate treatment, including during the antepartum period, is a component of good medical care. As in all cases, when considering treatments, the benefits should outweigh the risks. With Category A drugs, there are adequate, well-controlled studies in pregnant women that have not shown an increased risk of fetal abnormalities to the fetus in any trimester of pregnancy. With Category B, animal studies have revealed no evidence of harm to the fetus; however, there are no adequate and well-controlled studies in pregnant women or animal studies that have shown an adverse effect, but adequate and well-controlled studies in pregnant women have failed to demonstrate a risk to the fetus in any trimester. Category C drugs have animal studies that show an adverse effect and there are no adequate and well-controlled studies in humans, but potential benefits may warrant use of the drug in pregnant women despite potential risks. Category D drugs have adequate well-controlled or observational studies in pregnant women and are known risks to the fetus. Category X drugs should not be used in pregnancy, because adequate well-controlled or observational studies in animals or pregnant women have demonstrated positive evidence of fetal abnormalities or risks.

A 22-year-old G1P1 woman delivered her first baby five days ago after a prolonged labor and subsequent Cesarean section for arrest of cervical dilation at 7 cm. Fever was noted on postoperative day two and, despite broad spectrum antibiotics, she continues to have temperature spikes above 101.3° F (38.5° C). She is eating a normal diet and ambulating normally. On physical examination, her breasts have no erythema and nipples are intact. Her abdomen is soft, uterine fundus is firm and nontender, and her incision is healing without induration or erythema. She has normal lochia and her urinalysis is normal. Pelvic examination reveals a firm nontender uterus and no adnexal masses or tenderness. Which of the following is the most likely cause of her fevers? A. Mastitis B. Endometritis C. Ovarian abscess D. Cystitis E. Septic pelvic thrombophlebitis

C. Septic thrombophlebitis involves thrombosis of the venous system of the pelvis. Diagnosis is often one of exclusion of other causes, but sometimes a CT scan will reveal thrombosed veins. Treatment requires addition of anticoagulation to antibiotics and resolution of fevers is rapid. Anticoagulation treatment is short-term. Classic clinical findings for endometritis include fever and maternal tachycardia, uterine tenderness and no other localizing signs of infection. The clinical manifestations of cystitis include lower abdominal pain, frequency, urgency and dysuria. The clinical findings in patients with mastitis include fever, tenderness, induration and erythema of the affected breast.

29-year-old G1P0 woman at 42 weeks gestation presents in labor. She denies ruptured membranes. Her prenatal course was complicated by chronic hypertension. Her vital signs are: blood pressure 130/80; pulse 72; afebrile; fundal height 36 cm; and estimated fetal weight of 2100 gm. Cervix is dilated to 4 cm, 100% effaced, +1 station. The fetal heart rate tracing is shown below. What is the most likely diagnosis? (fetal heart rate min after peak of CTX) A. Normal fetal heart rate pattern B. Sinusoidal rhythm C. Late deceleration D. Variable decelerations E. Early decelerations

C. Late decelerations are a symmetric fall in the fetal heart rate, beginning at or after the peak of the uterine contraction and returning to baseline only after the contraction has ended. Late decelerations are associated with uteroplacental insufficiency. Variable decelerations show an acute fall in the FHR with a rapid down slope and a variable recovery phase. They are characteristically variable in duration, intensity, and timing, and may not bear a constant relationship to uterine contractions. Early decelerations are physiologic caused by fetal head compression during uterine contraction, resulting in vagal stimulation and slowing of the heart rate. This type of deceleration has a uniform shape, with a slow onset that coincides with the start of the contraction and a slow return to the baseline that coincides with the end of the contraction. Thus, it has the characteristic mirror image of the contraction. The true sinusoidal pattern is a regular, smooth, undulating form typical of a sine wave that occurs with a frequency of two to five cycles/minute and an amplitude range of five to 15 beats per minute. It is also characterized by a stable baseline heart rate of 120 to 160 beats per minute and absent beat-to-beat variability.

A 19-year-old G1P0 woman at 40 weeks gestation has an uncomplicated vaginal delivery followed by a brisk hemorrhage. Her past medical history is significant for steroid-dependent asthma. Her blood pressure is 110/70; pulse 84; and she is afebrile. Which of the following uterotonic agents should not be used in this patient? A. Intramuscular oxytocin B. Intravenous oxytocin C. Prostaglandin F2-alpha D. Prostaglandin E1 (Misoprostol) E. Methylergonovine

C. Methergine, prostaglandins and oxytocin are all uterotonics and used to increase uterine contractions and decrease uterine bleeding. Prostaglandin F2-alpha (Hemabate) is a potent smooth muscle constrictor, which also has a bronchio-constrictive effect. As such, it should be used with caution in any patient with a reported history of asthma. It is absolutely contraindicated in patients with poorly controlled or severe asthma. Prostaglandin E1 (Misoprostol), often used for cervical ripening and labor induction, is frequently used in oral or rectal suppository form for treatment of uterine atony, although not FDA approved for this use.

A 22-year-old G2P1 woman is at 42 weeks gestation dated by an ultrasound performed five weeks ago. Her cervix is long and closed. She does not report contractions and states there is good fetal movement. She would like to wait until she goes into labor spontaneously. Which of the following treatment options is optimal at this time? A. Allow the patient to go into labor spontaneously B. Perform an ultrasound to determine gestational age C. Perform a non-stress test (NST) and amniotic fluid index (AFI) twice a week, with induction of labor for a nonreactive NST or oligohydramnios. D. Patient should perform daily fetal movement counts and proceed with induction for decreased fetal movement. E. Perform daily biophysical profiles and deliver if 4 or less

C. Optimal management for the patient with an unfavorable cervix at an uncertain 42 weeks gestation is arguable. Given the uncertainty of her dates, it is reasonable to follow this patient with antepartum fetal testing, such as twice weekly non-stress tests with amniotic fluid index. The risk of fetal death is 1-2/1000 high-risk pregnancies with a reassuring NST, contraction stress test or biophysical profile. The addition of amniotic fluid assessment may improve the predictive value of a reactive NST and reduce the risk of antepartum fetal demise to even lower levels. Ultrasound for gestational age determination in the third trimester is not useful since the measurement error is +/- 3 weeks. Allowing spontaneous onset of labor is okay, but not without some type of antepartum surveillance testing.

A 28-year-old G3P2 woman presents in labor at 39 weeks gestation and delivers a 3500 gram infant spontaneously after oxytocin augmentation of labor. Thirty minutes later, the placenta has not delivered. Her past medical history is significant for leiomyoma uteri. Her prenatal course was uncomplicated. What is the most likely risk factor for retained placenta in this case? A. Placental abruption B. Labor augmentation C. Leiomyomas D. Multiparity E. Circumvallate placenta

C. Placental abruptions, labor augmentation, degree of parity and circumvallate placenta have no impact on the risk of retained placenta. The following are associated with retained placenta: prior Cesarean delivery, uterine leiomyomas, prior uterine curettage and succenturiate lobe of placenta.

A 35-year-old G1P0 woman at 30-weeks gestation is transferred from an outside hospital in preterm labor. Her cervix is 3 cm dilated, 50% effaced and the vertex is at 0 station. She is having contractions every five minutes and has no signs consistent with an intra-amniotic infection (chorioamnionitis). She was initially treated with terbutaline prior to her transfer. Which of the following side effects would you expect? A. Premature constriction of the ductus arteriosus B. Respiratory depression C. Tachycardia D. Tachypnea E. Headache

C. Terbutaline is a beta-adrenergic agent. Side effects include tachycardia, hypotension, anxiety and chest tightening or pain. Tachypnea and headaches are not usual side effects. The FDA made a formal announcement in 2011 warning against using terbutaline to stop preterm labor stating that terbutaline is both ineffective and dangerous if used for longer than 48 hours. The drug may still be used on a short-term basis in patients with active contractions, such as those being transferred to another hospital for tertiary care. Alternative tocolytic agents should be used for longer term treatment of preterm labor. Non-steroidal anti-inflammatory agents, such as indomethacin can be used as a tocolytic agent, but would have the side effect of premature closure of the ductus arteriosus if used beyond 32 weeks gestation. Magnesium sulfate can also be used as a tocolytic and has the potential side effect of respiratory depression.

A 28-year-old G1P1 woman presents to your office. She delivered four weeks ago and tearfully reports that she is not sleeping, feels anxious and has thoughts of jumping out her 15th floor window. What is the most likely diagnosis? A. Postpartum anxiety B. Postpartum blues C. Postpartum depression D. Postpartum psychosis E. Bipolar disorder

C. The patient is describing symptoms of depression. Symptoms such as mood changes, insomnia, phobias and irritability are more pronounced than with the "blues." She has not described any of the even more advanced psychotic symptoms of visual or auditory hallucinations.

A 29-year-old G1P0 woman is at 11 weeks gestation. She has a history of depression which has been well controlled with fluoxetine (Prozac). Although the medication is very helpful in controlling her depression, she is concerned about potential side effects on her neonate. Which of the following conditions in the neonate is associated with maternal use of Fluoxetine during pregnancy? A. Necrotizing enterocolitis B. Intracranial hemorrhage C. Agitation and poor feeding D. Temperature instability E. Persistent pulmonary hypertension

C. Third trimester maternal use of SSRIs including Fluoxetine has been associated with abnormal muscle movements (extrapyramidal signs or EPS) and withdrawal symptoms which may include agitation, abnormally increased or decreased muscle tone, tremor, sleepiness, severe difficulty breathing, and difficulty in feeding. In some newborns, the symptoms subside within hours or days and do not require specific treatment; other newborns may require longer hospital stays. SSRI use during pregnancy is not associated with newborn seizures, intracranial hemorrhage or temperature instability. The FDA has concluded that, given the conflicting results from different studies, it is premature to reach any conclusion about a possible link between SSRI use in pregnancy and persistent pulmonary hypertension.

A 32-year-old G1 is at 36 weeks gestation. Ultrasound reveals limited fetal growth over the past three weeks. Biometry is consistent with 30-5/7, EFW 1900 g, less than 10th percentile. Which of the following is LEAST likely to be associated with this pregnancy? A. Fetal demise B. Perinatal demise C. Polyhydramnios D. Meconium aspiration E. Polycythemia

C. This fetus had intrauterine growth restriction and, with the exception of polyhydramnios, all of the morbidities listed above may complicate intrauterine growth restriction. In general, the causes of polyhydramnios relate to amniotic fluid production (abnormalities of the fetal urinary tract) and removal (abnormalities of fetal swallowing and intestinal reabsorption of fluid). Some investigators report an increase in fetal urinary output when there is hyperglycemia and increased renal osmotic load, thus resulting in polyhydramnios. Abnormal fetal swallowing may be a result of a CNS or gastrointestinal tract abnormalities, such as anencephaly, esophageal or duodenal atresia, diaphragmatic hernia or primary muscular disease. Typically, polyhydramnios is not associated with asymmetric growth restriction (the most common form of IUGR), since an asymmetric growth pattern reflects poor uterine blood flow and limited substrate availability. In fact, oligohydramnios is frequently identified in pregnancies complicated by fetal growth restriction.

A 19-year-old G1P0 woman at 41 weeks with spontaneous rupture of membranes for 13 hours presented to labor and delivery. She had an uncomplicated prenatal course. Her vital signs are: blood pressure 120/70; pulse 72; afebrile; fundal height 36 cm; and estimated fetal weight of 2700 gm. Cervix is dilated to 4 cm, 100% effaced, + 1 station. What does the fetal heart rate tracing seen below show? (fetal heart rate trough min prior to max of CTX) A. Late deceleration B. Variable decelerations C. Early decelerations D. Sinusoidal rhythm E. Normal fetal heart rate pattern

C. Early decelerations are physiologic caused by fetal head compression during uterine contractions, resulting in vagal stimulation and slowing of the heart rate. This type of deceleration has a uniform shape, with a slow onset that coincides with the start of the contraction and a slow return to the baseline that coincides with the end of the contraction. Thus, it has the characteristic mirror image of the contraction. A late deceleration is a symmetric fall in the fetal heart rate, beginning at or after the peak of the uterine contraction and returning to baseline only after the contraction has ended. Late decelerations are associated with uteroplacental insufficiency. Variable decelerations show an acute fall in the FHR with a rapid down slope and a variable recovery phase. They are characteristically variable in duration, intensity, and timing, and may not bear a constant relationship to uterine contractions. The true sinusoidal pattern is a regular, smooth, undulating form typical of a sine wave that occurs with a frequency of two to five cycles/minute and an amplitude range of five to 15 beats per minute. It is also characterized by a stable baseline heart rate of 120 to 160 beats per minute and absent beat-to-beat variability.

A 19-year-old G1P0 woman at 28 weeks gestation comes to labor and delivery because of the onset of contractions. The patient describes the contractions as progressively becoming more painful, each lasting 40 seconds and now occurring every five minutes. She reports good fetal movement and does not have any bleeding or leakage of fluid. On evaluation in triage, it is noted that she is having regular contractions, approximately every five minutes, has intact membranes and her cervical exam is 3 cm dilated and 50% effaced. What is the most frequent cause of this condition? A. Dehydration B. Fetal anomalies C. Idiopathic D. Uterine fibroids E. Cervical incompetence

C. In most cases, preterm labor is idiopathic (i.e. no cause can be identified). Dehydration and uterine distortion (from uterine fibroids or structural malformations) can be associated with preterm labor. In some cases, preterm labor is due to iatrogenic causes; for example, when a physician induces a preterm patient who has severe preeclampsia. Fetal anomalies typically do not cause preterm labor. Cervical incompetence is usually diagnosed earlier in the second trimester and is associated with painless cervical dilation.

A 28-year-old G1 woman at 31 weeks gestation presents with complaints of fluid leaking from the vagina. Preterm premature rupture of membranes is diagnosed. The patient has mild uterine tenderness concerning for early chorioamnionitis. An amniocentesis is performed. Which of the following amniotic fluid results is indicative of an intra-amniotic infection? A. Presence of leukocytes B. Low Interleukin-6 C. Amniotic glucose less than 20 mg/dl D. Elevated level of bilirubin E. Lecithin/sphingomyelin (L/S) ratio <2

C. In some cases of preterm rupture of the membranes, amniocentesis may be performed to detect intra-amniotic infection. The presence of amniotic leukocytes has the lowest predictive value for the diagnosis of chorioamnionitis. Interleukin-6 would be increased in the setting of chorioamnionitis. A low amniotic fluid glucose is an indication of intra-amniotic infection. L/S ratio is a marker for fetal lung maturity.

A 40-year-old G6P5 woman delivered a 5020 grams live born male infant vaginally 20 minutes ago. The nurse notes that the patient is having ongoing bleeding that she estimates to be 1000 cc. Her past medical history is significant for chronic hypertension and asthma. She has received oxytocin and misoprostol. Her examination reveals a boggy uterus. Which of the following is the next step in management of her bleeding? A. Uterine artery embolization B. B-Lynch compression stitch C. Bakri balloon placement D. Uterine artery ligation E. Hysterectomy

C. This patient has uterine atony, which accounts for about 80-90% of all postpartum hemorrhages. Risk factors for PPH include uterine over distension (polyhydramnios, macrosomia, and multiple gestation), prolonged labor, chorioamnionitis, and grandmultiparity. This patient has already received uterotonic agents and while additional agents are available, because of her hypertension and asthma both methergine and prostaglandin F2 alpha are contraindicated. All of the choices listed are treatments, the least invasive treatment is placement of the Bakri balloon, which is a device placed into the uterus with a balloon that is filled with up to 500 cc of sterile fluid. This places pressure on the inside of the uterus. Uterine artery ligation, B-lynch compression stitch and hysterectomy all require a laparotomy and should be reserved for recalcitrant cases. Uterine artery embolization requires placement of embolization catheters as well as interventional radiology.

A 28-year-old G3P3 woman experiences profuse vaginal bleeding of 700 cc in one hour following an uncomplicated spontaneous vaginal delivery of a 4150 gram infant. The placenta delivered spontaneously without difficulty. Prior obstetric history is notable for a previous low transverse Cesarean delivery, secondary to transverse fetal lie. The patient had no antenatal complications. Which of the following is the most likely cause of this patient's hemorrhage? A. Vaginal or cervical lacerations B. Uterine inversion C. Uterine atony D. Uterine dehiscence E. Uterine rupture

C. Uterine atony is the most common cause of postpartum hemorrhage. Risk factors for uterine atony include precipitous labor, multiparity, general anesthesia, oxytocin use in labor, prolonged labor, macrosomia, hydramnios, twins and chorioamnionitis. Patients at risk for genital tract lacerations are those who have a precipitous labor, macrosomia or who have an instrument-assisted delivery or manipulative delivery (i.e. breech extraction). Factors that lead to an over-distended uterus are risk factors for uterine inversion. Grand multiparity, multiple gestation, polyhydramnios and macrosomia are all risk factors. The most common etiology of uterine inversion, however, is excessive (iatrogenic) traction on the umbilical cord during the third stage of delivery. Although the patient is at risk for uterine dehiscence/uterine rupture because of her history of a prior Cesarean delivery, these are infrequent occurrences so the most likely cause of postpartum hemorrhage in this patient is uterine atony.

A 25-year-old G1P0 woman at 29 weeks gestation presents to labor and delivery complaining of contractions every 3 minutes. On exam, her blood pressure is 120/70; heart rate is 100; temperature is 100.7°F (38.2°C); and fetal heart tones are in the 160's. The fetal heart rate tracing is showing deep variable decelerations, but is otherwise is reassuring. Her pelvic examination reveals gross rupture of membranes and her cervix is dilated to 6 cm and ultrasound reveals the baby is in the breech presentation. What of the following explains the variable decelerations? A. Chorioamnionitis B. Labor C. Premature Rupture of Membranes D. Breech presentation E. Gestational age

C. Variable decelerations are a result of cord compression. With cord compression, initially there is a short increase in fetal heart rate followed by an abrupt drop in heart rate. As the compression is released, the fetal heart rate returns to baseline, frequently with a brief, slight increase. Variable decelerations can occur at any point. This patient has PPROM and a frequent cause of cord compression can be lack of amniotic fluid. Chorioamnionitis is an infection of the amniotic fluid and chorion that explains this patient's fever, but is not a cause of variable decelerations. Breech presentation, and gestational age do not cause variable decelerations.

A 45-year-old G4P3 woman presents with vaginal bleeding. Last week, she performed a home pregnancy test that was positive. She thinks her last menstrual period was four months ago. The last time she saw her doctor was eight years ago, with the birth of her last child. She has no serious medical problems, has smoked a pack of cigarettes a day since the age of 20, occasionally has a beer and does not exercise. Abdominal examination reveals a soft abdomen and the fundus palpable just below the umbilicus. Pelvic ultrasound reveals a fundal placenta and a fetus measuring 18 weeks with normal cardiac activity. Vaginal examination reveals a 3-centimeter lesion on the posterior lip of the cervix. It easily bleeds with palpation and is hard in consistency. Which of the following is the most likely cause of the bleeding? A. Cervicitis B. Cervical polyp C. Endometrial polyp D. Cervical cancer E. Nabothian cyst

D. Cervical cancer can unfortunately complicate pregnancies and presents with bleeding. She is at risk due to lack of screening as well as her history of smoking. Other causes of bleeding need to be ruled out such as cervical incompetence, infection or trauma. Treatment for cervical cancer during pregnancy requires difficult decisions that consider the stage of cancer, appropriate therapy, maternal welfare and fetal welfare. Cervical polyps occur during pregnancy and can be a cause of bleeding, but are typically soft and not hard or nodular on examination. Nabothian cysts are very common, but do not typically cause bleeding.

A 24-year-old G2P1 woman is undergoing a Cesarean delivery for placental abruption. She presented to labor and delivery with severe abdominal pain and heavy vaginal bleeding. The fetus was delivered uneventfully. The placenta delivered with a significant clot attached to the maternal surface. The patient continues to bleed from the placental bed. Estimated blood loss is 1500 ml. The operative team decides to give her fresh frozen plasma (FFP) to replace which of the following components? A. Platelets B. Von Willebrand's factor C. Red blood cells D. Fibrinogen E. Factor X

D. Correcting coagulation deficiencies requires replacing all necessary blood components. Fresh frozen plasma contains fibrinogen, as well as clotting factors V and VIII. Cryoprecipitate contains fibrinogen, factor VIII and von Willebrand's factor. Neither of these preparations contains red blood cells or platelets, which must be given separately.

A 23-year-old G1P1 woman is five days post-operative from a Cesarean section for arrest of labor at 7 cm. She now complains of minimal abdominal pain and drainage from the right side of the incision. Lochia is normal and she has no urinary complaints. Her vital signs are normal and she is afebrile. On physical exam, her lung and cardiac examinations are normal. Her abdomen and uterine fundus are nontender. Her Pfannenstiel incision has tenderness extending 3 cm from the incision and there is serous, bloody drainage coming from the right side. What is the next best step in the management of this patient? A. Initiate intravenous antibiotics B. Initiate oral antibiotics C. Occlusive dressing to the wound D. Open drainage of wound E. Tropical antibiotics to the wound

D. Mixed bacteria originating from the skin, uterus and vagina cause wound infections after a Cesarean section. Prior to establishing a diagnosis of surgical site infection, evaluation requires opening the wound, checking for fascial dehiscence, drainage and assessment of the fluid. Packing the wound until it has healed from the base of the wound facilitates the healing process. Broad spectrum antibiotics are indicated if you suspect cellulitis or abscess.

A 32-year-old G2P2 woman delivered five days ago by uncomplicated vaginal delivery. Her postpartum course thus far has been unremarkable and she is breastfeeding without difficulty. She woke up in the middle of the night with intense upper abdominal pain and chills. She admits that she has had pain like this before, but never this severe. Her vital signs reveal blood pressure 120/70; pulse 110; and temperature 101.8° F (38.8° C). On physical examination, she has abdominal pain located in the right upper quadrant with rebound tenderness. Her uterine fundus is well below the umbilicus and nontender. Her lochia is normal. Laboratory tests reveal mild anemia, a slightly elevated white count and slightly elevated liver function tests. What is the most likely etiology of her pain? A. Endomyometritis B. Ruptured ovarian abscess C. Hepatitis D. Cholecystitis E. Appendicitis

D. Non-pregnancy related conditions must be considered when evaluating women in the postpartum period. Pregnancy puts women at risk for cholelithiasis and, therefore, cholecystitis. Classic symptoms include nausea, vomiting, dyspepsia and upper abdominal pain after eating fatty foods. Treatment would be dependent on the severity of symptoms, but often involves cholecystectomy that is usually performed laparoscopically. Classic clinical findings for endomyometritis include fever and maternal tachycardia, uterine tenderness and no other localizing signs of infection. This patient is unlikely to have an ovarian cyst. Appendicitis presents with nausea, vomiting, anorexia and abdominal pain. Although hepatitis may be associated with elevated liver function tests, patients don't typically present with acute severe abdominal pain.

A 23-year-old G1P0 woman presents in labor at term. Her prenatal course was uncomplicated. She delivers a 3500 gram infant spontaneously after oxytocin augmentation of labor. Immediately postpartum, there is excessive bleeding greater than 2000 cc. There are no lacerations and the uterus is found to be boggy. Her blood pressure is 90/40; pulse is 120. Conservative and medical management have failed and you proceed with an exploratory laparotomy. Which of the following is the most appropriate next step in the management of this patient? A. Cervical artery ligation B. Ovarian artery ligation C. External iliac artery ligation D. B-Lynch suture E. Hysterectomy

D. A uterine compression suture such as a B-Lynch has been shown to be effective in the management of unresponsive uterine atony. Ligation of a number of pelvic vessels can lead to reduction in the vascular pressure in the pelvis thus controlling hemorrhage. This is especially true with internal iliac artery (hypogastric artery) ligation. However, ligation of the ovarian arteries should not be undertaken as a primary approach. Ligation of the external iliac artery results in devascularization of the leg and, therefore, should not be performed. If these more conservative maneuvers fail, hysterectomy may be necessary but should be a last resort considering the age and parity of the patient.

A 23-year-old G1P1 woman delivered vaginally at 42-weeks gestation after a prolonged induction of labor. She had an epidural with an indwelling catheter for 36 hours and three IV sites for her intravenous medications. She now complains of lower abdominal pain, frequency and dysuria. Her vital signs are: temperature 98.6° F (37° C); pulse 70; blood pressure 100/60; and respirations 12. On examination, her lungs are clear, cardiac exam is normal, abdomen is soft, uterine fundus is firm and nontender, and she has mild suprapubic tenderness. Which of the following organisms is most likely causing her discomfort? A. Group A streptococcus B. Proteus mirabilis C. Klebsiella pneumoniae D. Escherichia coli E. Group B Streptococcus

D. Acute cystitis is a common complication after vaginal delivery and the risk increases with the use of an indwelling catheter. The most common cause of acute cystitis infection is gram-negative bacteria. The major pathogens are E. coli (75%), P. mirabilis (8%), K. pneumoniae (20%), S. faecalis (<5%), and S. agalactiae.

A 32-year-old G0 woman presents for preconception visit. She has regular periods reports tension, depressed mood and decreased productivity towards the end of each cycle. She is otherwise healthy and maintains a high-profile job. Her past medical history is benign and she denies prior psychiatric problems. She denies smoking and drinks alcohol socially. What is the next best step in the management of this patient? A. Reassure her that her monthly symptoms are normal B. Initiate anti-depressant therapy C. Psychiatry consult D. Ascertain the timing of her symptoms each month E. Initiate psychotherapy

D. Ascertaining the timing of her symptoms each month is an important first step in establishing the proper diagnosis. Symptoms of Premenstrual Dysphoric Disorder occur in the luteal phase and are absent in the beginning of the follicular phase. It is therefore important to document the timing of symptoms each month when considering a diagnosis of Premenstrual Dysphoric Disorder. Additionally, it is important to ascertain that these symptoms are not an exacerbation of an underlying psychiatric disorder before initiating therapy as this potentially can have more consequences during her pregnancy and postpartum period.

A 23-year-old G1P1 woman delivered her first baby two days ago after an uncomplicated labor and vaginal delivery. She wants to breastfeed and has been working with the lactation team. Prior to discharge, her temperature was 100.4° F (38° C) and other vitals were normal. She denies urinary frequency or dysuria and her lochia is mild without odor. On examination, her lungs are clear, cardiac exam normal, and abdomen and uterine fundus are nontender. Her breasts are firm and tender throughout, without erythema, and nipples are intact. Which of the following is the most likely cause of her fever? A. Endomyometritis B. Septic pelvic thrombophlebitis C. Mastitis D. Breast engorgement E. Vaginitis

D. Breast engorgement is an exaggerated response to the lymphatic and venous congestion associated with lactation. Milk "let-down" generally occurs on postpartum day two or three. If the baby is not feeding well, the breast can become engorged, which can cause a low-grade fever. Lactating women are encouraged to feed their baby frequently, and use a breast pump to prevent painful engorgement and mastitis. Postpartum fever differential includes endometritis, cystitis and mastitis. These are easy distinguished, based on clinical findings. Vaginitis is not accompanied by fever. Septic pelvic thrombophlebitis is a rare condition and characterized by high fever not responsive to antibiotics and is a diagnosis of exclusion.

A 36-year-old G1 with type 1 diabetes is diagnosed with intrauterine growth restriction at 33 weeks gestation. What is the most appropriate next step in management? A. Amniocentesis B. Immediate delivery C. Weekly ultrasounds to assess fetal growth D. Antenatal testing of fetal well-being E. Observation

D. Once intrauterine growth restriction is detected, the fetus needs to be evaluated periodically for evidence of well-being until delivery is deemed necessary. This will result in once or twice weekly testing, depending on the modality of assessment that is being used. Testing includes: non-stress test (NST), where the fetal heart beat is recorded over a period of at least 30 minutes while looking for accelerations with fetal movement, and the biophysical profile, which includes an ultrasound evaluation of fetal movement, fetal tone, amniotic fluid and breathing. NSTs should be performed twice weekly with at least a weekly AFI. The BPP may be performed weekly. Ultrasound for fetal growth is not useful if more frequent than every two weeks. An amniocentesis for fetal lung maturity can be considered at more advanced gestational age.

A 32-year-old G2P1 woman is at 42 weeks gestation. Her prenatal course was uncomplicated and she had a first trimester ultrasound confirming dates. Her cervix is 4 cm dilated and 100% effaced. She does not report contractions and states there is good fetal movement. What is the next best step in the management of this patient? A. Ultrasound to assess amniotic fluid volume B. Twice weekly non-stress test (NST) and amniotic fluid index (AFI) C. Daily biophysical profiles D. Admit for induction E. Ultrasound to assess fetal growth

D. Optimal management for the patient with a favorable cervix at greater than or equal to 41 weeks gestation is delivery. Her dilation and effacement make it likely her induction will be successful. Induction of labor in a patient with an unfavorable cervix increases the risk of Cesarean section significantly, compared to a patient who goes into spontaneous labor. It is not advisable to follow a patient who is >42 weeks with antepartum fetal testing, such as twice weekly non-stress tests with amniotic fluid index, if the gestational age is certain. Performing an ultrasound to assess fetal growth and/or amniotic fluid volume should not change the management plan which should be induction of labor at this gestational age.

A 30-year-old G4P3 woman at 24 weeks gestation is found to have an anterior placenta previa. She has a history of three prior Cesarean deliveries. What is the most likely serious complication that can lead to obstetric hemorrhage in this woman? A. Placental abruption B. Uterine dehiscence prior to labor C. Uterine inversion D. Placenta accreta E. Uterine atony

D. Placental abruption and uterine atony are both common, but, in the presence of a low-lying anterior placenta in a patient with a history of multiple Cesarean births, the diagnosis of the placenta accreta must be entertained. Placenta accreta is an abnormally firm attachment of the placenta to the uterine wall. The incidence of placenta accreta may be increasing because of the rise in the number of women with previous Cesarean deliveries. This is a serious obstetric complication leading to retained placenta and severe postpartum hemorrhage. Hysterectomy is frequently required due to intractable hemorrhage at delivery.

A 20-year-old G2P1 woman is at 41 weeks gestation. Her prenatal course and past medical history are unremarkable. She has not had any complications with her pregnancy and fetal surveillance is reassuring. Which of the following complications is most likely to occur in this pregnancy? A. Preeclampsia B. Retained placenta C. Postpartum hemorrhage D. Macrosomia E. Placenta abruption

D. Postterm pregnancies are associated with macrosomia, oligohydramnios, meconium aspiration, uteroplacental insufficiency and dysmaturity. Although postterm infants are larger than term infants and have an increased incidence of fetal macrosomia, there is no evidence to support induction of labor as a preventive measure for macrosomia in these cases. There is no associated risk for preeclampsia in postterm gestations.

A 22-year-old G2P1 woman presents for prenatal care at approximately 10 weeks gestation. Her first pregnancy was complicated by preterm premature rupture of the membranes at 28 weeks gestation. Which of the following interventions could reduce the risk of preterm premature rupture of the membranes during this pregnancy? A. Bedrest B. Placement of a cerclage C. Placement of a Tertbutaline pump D. 17 alpha-hydroxyprogesterone E. Nifedipine

D. Premature rupture of the membranes occurs in approximately 10-15% of all pregnancies. Preterm premature rupture of the membranes between 16 and 26 weeks gestation is identified in 1% of pregnancies. Preterm premature rupture of the membranes occurs in 1/3 of all preterm deliveries. The reported recurrence rate for preterm premature rupture of the membranes is approximately 32% when it occurred in the index pregnancy. Bedrest and tocolytics have not been shown to reduce the risk for PPROM, and may have detrimental effects to the mother. A cerclage may be indicated for patients with a history of an incompetent cervix. 17 alpha-hydroxyprogesterone has been shown to reduce the risk of premature labor. 17 alpha-hydroxyprogesterone is administered weekly (starting between 16-20 weeks) until 36 weeks gestation.

A 22-year-old G1P0 woman presents at 42 weeks gestation. Her cervix is long and closed. She does not report contractions and states there is good fetal movement. You discuss the benefits of induction at this time versus waiting until she goes into labor spontaneously. She agrees to proceed with an induction. Which of the following is the best next step in the management of this patient? A. Artificial rupture of membranes B. Membrane stripping C. Oxytocin infusion D. Prostaglandin E1 tablet E. RU486 (progesterone antagonist)

D. Prostaglandins applied locally are the most commonly-used cervical ripening agents. RU486 is not used for cervical ripening. Membrane stripping (digital separation of chorioamnion from lower uterine segment) and artificial rupture of membranes cannot be performed in a patient with a closed cervix. The American College of Obstetricians and Gynecologists (ACOG) recommendations for the management of postterm pregnancy includes patient records fetal kick counts and fetal surveillance using one of the following: NST, CST, biophysical profile and delivery for nonreassuring testing. If the patient has a favorable cervix, induce at 42 weeks and, if the cervix is unfavorable, use cervical ripening agents.

A 29-year-old G2P1 woman at 39 weeks gestation presents in early labor after spontaneous rupture of the fetal membranes. Thirty minutes after arrival, she delivers a 2650 gram male infant. A globular pale mass appears at the introitus when attempting to deliver the placenta. Her blood pressure is 90/60; pulse 104; and temperature is 98.6°F (37.0°C). What is the most likely etiology for this event in this patient? A. Multiparity B. Twin gestation C. Leiomyoma D. Uterine inversion E. Rapid labor

D. Uterine inversion is an uncommon etiology of postpartum hemorrhage. Factors that lead to an over-distended uterus are risk factors for uterine inversion. Grand multiparity, multiple gestation, polyhydramnios and macrosomia are all risk factors. The most common risk factor, however, is excessive (iatrogenic) traction on the umbilical cord during the third stage of delivery. Although leiomyomas may spontaneously prolapse, it is unlikely during the peripartum period.

A 34-year-old chronic hypertensive G1 comes to see you for a consultation at 34 weeks for size less than dates. Her prenatal course has been uncomplicated and the genetic amniocentesis obtained at 15 weeks revealed a normal male. Biometrics today reveal a biparietal diameter consistent with 33 weeks, abdominal circumference of 28 weeks, EFW 1600 g, less than 10th percentile, and an amniotic fluid index of 6. What is the most likely cause of fetal growth restriction in this patient? A. Chromosomal abnormality B. Fetal infection with Rubella C. Fetal infection with cytomegalovirus (CMV) D. Uteroplacental insufficiency E. Maternal infection with Varicella

D. Uteroplacental insufficiency can lead to asymmetric growth restriction. Asymmetric growth restricted infants typically have a normal length, but their weight is below normal. On ultrasound, there is a head-sparing effect, meaning that the head/brain is spared of the reduced blood flow that is a result of uteroplacental insufficiency. Thus, the fetal abdomen measures below normal and the head remains very close to normal. There is an asymmetrical growth pattern that is usually detected during the third trimester and reflects uteroplacental insufficiency. Symmetric fetal growth restriction indicates that all fetal measurements are below normal. As a general rule, such a finding indicates an intrinsic growth failure or an "early event" secondary to one or more organ system anomalies, fetal aneuploidy or chronic intrauterine infection. Infectious diseases are known to cause IUGR, but the number of organisms is poorly defined. There is sufficient evidence to show a causal relationship between rubella and CMV infections and fetal growth restriction. Other viruses to consider are syphilis and varicella. The protozoan toxoplasmosis results in IUGR as well. There are no bacteria known to cause IUGR. Symmetrical growth restriction is usually detected in the mid-trimester of pregnancy.

A 19-year-old G1P0 woman at 42 weeks gestation presents to labor and delivery with spontaneous rupture of membranes for 13 hours and spontaneous onset of labor. Her vital signs are: blood pressure 120/70; pulse 72; afebrile; fundal height 36 cm; and estimated fetal weight of 2700 gm. Cervix is dilated to 4 cm, 100% effaced, +1 station. Which statement best describes the tracing seen below? (fetal heart rate baseline 130s with high moderate variability and decels. no relationship to CTX) A. Normal fetal heart rate pattern B. Sinusoidal rhythm C. Late deceleration D. Variable deceleration E. Early deceleration

D. Variable decelerations show an acute fall in the FHR, with a rapid down slope and a variable recovery phase. They are characteristically variable in duration, intensity, and timing, and may not bear a constant relationship to uterine contractions. They are typically associated with cord compression, especially in the setting of low amniotic fluid volume. Early decelerations are physiologic caused by fetal head compression during uterine contraction, resulting in vagal stimulation and slowing of the heart rate. This type of deceleration has a uniform shape, with a slow onset that coincides with the start of the contraction and a slow return to the baseline that coincides with the end of the contraction. Thus, it has the characteristic mirror image of the contraction. A late deceleration is a symmetric fall in the fetal heart rate, beginning at or after the peak of the uterine contraction and returning to baseline only after the contraction has ended. Late decelerations are associated with uteroplacental insufficiency. The true sinusoidal pattern is a regular, smooth, undulating form typical of a sine wave that occurs with a frequency of two to five cycles/minute and an amplitude range of five to 15 beats per minute. It is also characterized by a stable baseline heart rate of 120 to 160 beats per minute and absent beat-to-beat variability.

A 29-year-old G1P0 woman presents at 31 weeks gestation with preterm rupture of membranes six hours ago. She notes that for the last hour she has had some occasional contractions. Her prenatal course has been uncomplicated and she takes prenatal vitamins and iron. She denies substance abuse, smoking or alcohol use. Her blood pressure is 110/70; pulse 84; temperature 98.6°F (37.0°C). What is the role of tocolysis in this patient? A. Prevent delivery B. Delay delivery until fetal lung maturity is reached C. Delay delivery for one week D. Delay delivery in order to administer steroids E. Contraindicated

D. While the role of tocolysis in the setting of preterm rupture of membranes is controversial, it may be appropriate in limited settings. Tocolysis may be administered in an attempt to prolong the interval to delivery to gain time for steroids to obtain maximum benefit for the fetus. The risks of chorioamnionitis with continuing tocolytics beyond 48 hours outweighs the benefit of awaiting lung maturity. This may be reasonable in women without evidence of infection or advanced preterm labor. Admittedly, the likelihood of success in this setting is relatively poor, but the potential benefit to the fetus probably outweighs any maternal complication from tocolysis.

32-year-old G1P1 is status post uncomplicated normal spontaneous vaginal delivery. She is taking sertraline (Zoloft), a selective serotonin uptake inhibitor (SSRI) as an antidepressant and wants to breastfeed. What is the next best step in management of this patient? A. Decrease her SSRI dose by 50%, since these drugs are concentrated in the breast milk B. Consult psychiatry about changing medications and discard the expressed milk in the meantime C. Discontinue the medications so she can breastfeed D. Increase her SSRI dose, since these drugs are not concentrated in the breast milk and she is at great risk for postpartum depression E. Continue the medications, since there is negligible risk for the newborn

E. Breastfeeding is beneficial to both mother and infant. Current recommendations state that SSRI medications can be safely used during lactation. Several studies show that SSRIs are secreted in breast milk, however no detectable levels of the drug were found in the infants' serum. In addition, no adverse effects were noted in the infants by either their parents or pediatricians following the infants.

A 32-year-old G1 is seeing you in consultation at 35 weeks gestation. Ultrasound reveals limited fetal growth over the past three weeks. Biometry is consistent with 30-5/7, EFW 1900 g, less than 10th percentile. You counsel her about short and long-term complications for her baby. This fetus is at increased risk for all of the following adult disorders EXCEPT: A. Cardiovascular disease B. Chronic hypertension C. Chronic obstructive lung disease D. Diabetes E. Osteoporosis

E. Epidemiologic studies indicate that fetal growth restriction is a significant risk factor for the subsequent development of cardiovascular disease, chronic hypertension, chronic obstructive lung disease and diabetes. Researchers suggest that the phenomenon of programming may be operable and that an adverse fetal environment during a critical period of fetal growth helps to promote these adult diseases. Osteoporosis risk factors include family history, slender body composition, prior history of osteoporosis, Asian and Caucasian ethnicity, alcohol consumption, smoking, sedentary lifestyle, excess thyroid or corticosteroids and use of anticonvulsant medications.

A 19-year-old G3P0 woman at 39 weeks gestation with spontaneous rupture of membranes for 13 hours presented to labor and delivery. Her vital signs are: blood pressure 120/70; pulse 72; afebrile; fundal height 36 cm; and estimated fetal weight of 2700 gm. Cervix is dilated to 1 cm, 50% effaced, -2 station. Which statement best describes the tracing seen below? (fetal baseline in 170s, flattish line til end then irregular decel; CTX regular) A. Normal fetal heart rate with good variability and regular contractions B. Fetal tachycardia with good variability and regular contractions C. Normal fetal heart rate with poor variability and irregular contractions D. Fetal tachycardia with poor variability and regular contractions E. Normal fetal heart rate with good variability and irregular contractions

D. The baseline fetal heart rate is >160 with no accelerations or variability. There are regular contractions. Prolonged periods of fetal tachycardia are frequently found with maternal fever or chorioamnionitis.

A 22-year-old G2P1 woman has a history of a previous postterm pregnancy. She delivered a 3500 g healthy male infant at 42-½ weeks gestation via a Cesarean section secondary to fetal distress. She is currently 15 weeks pregnant, based on an irregular last menstrual period. What is the most appropriate management at this time? A. Plan for a repeat Cesarean section at 38 completed weeks B. Schedule for a repeat Cesarean section if she does not go into spontaneous labor by 40 weeks gestation C. Plan to admit the patient for an induction of labor (a VBAC) if she does not go into spontaneous labor by 41 weeks gestation D. Obtain a fetal ultrasound to date the pregnancy E. Start weekly non-stress tests and amniotic fluid indexes at 40 weeks gestation and proceed with either induction of labor or Cesarean section for a nonreactive non-stress test or oligohydramnios or if patient has not gone into spontaneous labor by 41 weeks gestation

D. The diagnosis of postterm pregnancy is based on the establishment of an accurate gestational age. In a patient with irregular menses, it is important to obtain an ultrasound prior to 20 weeks to accurately date the pregnancy. It is reasonable to allow a patient with reassuring fetal surveillance to go past 41 weeks gestation if her gestational age is accurately known. However, because of a prior Cesarean birth, consideration should be given to delivery before 41 weeks. Approximately 50% of patients with a history of a postterm pregnancy will experience prolonged pregnancy with the next gestation. There is currently no indication to begin antenatal testing at 40 weeks in this pregnancy.

A 22-year-old G1P1 woman delivered her first baby five days ago after a prolonged labor and subsequent Cesarean section for arrest of cervical dilation at 7 cm. Fever was noted on postoperative day two and, despite intravenous broad spectrum antibiotics, she continues to have temperature spikes above 101.3° F (38.5° C). She is eating a normal diet and ambulating normally. On physical examination, her breasts have no erythema and nipples are intact. Her abdomen is soft, uterine fundus is firm and nontender, and her incision is healing without induration or erythema. She has normal lochia and her urinalysis is normal. Pelvic examination reveals a firm nontender uterus and no adnexal masses or tenderness. Which of the following treatments is indicated for this patient? A. Addition of antifungal therapy B. Addition of oral antibiotic therapy C. Addition of antiviral therapy D. Surgical exploration E. Heparin anticoagulation

E. Septic thrombophlebitis involves thrombosis of the venous system of the pelvis. Diagnosis is often one of exclusion of other causes, but sometimes a CT scan will reveal thrombosed veins. Treatment requires addition of anticoagulation to antibiotics and resolution of fevers is generally rapid. Anticoagulation treatment is short-term. The addition of oral antibiotics has no extra benefit on a patient who is already on broad spectrum IV antibiotics. She has no evidence of fungal or viral infections, so therapy for these is not indicated. There is also no indication she needs surgery.

A 36-year-old G1 began prenatal care at eight weeks gestation. At that time, the gestational age was confirmed by a transvaginal ultrasound. She is now at 36 weeks gestation. Her previous medical history reveals hypertension for eight years and class F diabetes for five years (baseline proteinuria = 1 g). She smokes one half-pack of cigarettes per day. On examination at 32 weeks gestation, her fundal height was 29 cm. At 33 weeks, biometry was consistent with 31-3/7, EFW 1827g, 25th percentile. Today, ultrasound reveals limited fetal growth over the past three weeks. Biometry is consistent with 31-5/7, EFW 1900 g, <10th percentile. What is the most likely etiology of the intrauterine growth restriction in this case? A. Genetic factors B. Congenital anomaly C. Tobacco use D. Uteroplacental insufficiency E. Perinatal infection

D. There is substantial evidence from experimental animal studies that suggests that alterations in uteroplacental perfusion affect the growth and status of the fetus, as well as the placenta. This patient has significant medical diseases that are affecting her vasculature and, ultimately, limiting the substrate availability to the fetus with resultant uteroplacental insufficiency. The vascular disease is evidenced by retinopathy and proteinuria. The other choices above may all result in fetal growth restriction; however, they are not the most likely etiology in this clinical scenario.

A 33-year-old G1P0 woman at 38 weeks gestation with pregnancy complicated by type 1 diabetes was admitted for induction due to oligohydramnios. She received Cervidil (prostaglandin E2) overnight and her cervix was noted to be 3 cm dilated in the morning so oxytocin was started. After three hours on oxytocin induction, fetal heart rate was noted to be in the 160s with minimal variability and late decelerations despite resuscitation with oxygen, fluids and left lateral position. Thirty minutes after discontinuing the oxytocin, she continued to have contractions every three to four minutes with late decelerations. Her blood pressure was noted to be 138/88 and her pulse was 110. Her cervical exam was noted to be 4 cm dilated. What is the most appropriate next step in the management of this patient? A. Perform a biophysical profile B. Begin amnioinfusion C. Administer terbutaline D. Proceed with a Cesarean section E. Restart the oxytocin

D. This fetus is clearly not tolerating labor. Unfortunately, there is no good way to assess fetal status at this point. A biophysical profile is not of any value in labor. Amnioinfusion may be used for repetitive variable decelerations and not for recurrent lates. The presence of late decelerations in a patient with diabetes and oligohydramnios is not reassuring and unlikely to recover. Although terbutaline may slow down the contractions, it is not recommended in a patient whose heart rate is 110.

A 20-year-old G2P1 woman at 28 weeks gestation presents to labor and delivery with contractions every four minutes. On physical examination, her vital signs are: temperature 101°F (38.3°C); heart rate 120; respiratory rate 18; and blood pressure 110/65. Her uterine fundus is tender and the rest of the physical exam is normal. Her cervix is dilated 1 cm and is 50% effaced. The fetus is in vertex presentation. Fetal heart tones are in the 150s with a Category I tracing. Her white blood cell count (WBC) is 18,000/mcL. Which of the following is the most appropriate next step in the management of this patient? A. Observation B. Tocolysis C. Contraction stress test D. Labor induction E. Cesarean section

D. This patient has a fever, a tender fundus, and elevated white blood cell count, which are concerning for an intra-amniotic infection. Delivery is warranted and in the case of reassuring heart tones, there are no contraindications for labor induction and a Cesarean section is not indicated at this time. Tocolytics should not be used in the case of an intra-amniotic infection. Conservative management with observation would delay diagnosis and would not be appropriate. A contraction stress test is not indicated since the patient is already contracting with reassuring fetal heart tones.

A 29-year-old G1P0 woman at 28 weeks gestation presents with preterm labor. She reports having contractions every 2 minutes and describes them as painful. On exam, her blood pressure is 130/70; pulse 92; and is afebrile. Her cervix is dilated to 2 cm/50%effaced/-4 station. The fetus has a Category I tracing with contractions every 2 minutes. She is admitted and started on nifedipine and betamethasone. Which of the following medications is also indicated for this patient? A. Terbutaline B. Prostaglandin E1 C. Prostaglandin E2 D. Ampicillin E. No additional medications required

D. This patient is in preterm labor. Ampicillin is indicated for this patient as her Group B Strep status is unknown and should be continued until a culture result is negative or her labor stops. Nifedipine is a tocolytic used to delay the progression of labor to allow for the benefit of betamethasone to hasten pulmonary maturation. Both prostaglandin E1 and E2 are uterotonic agents and would likely increase the rate of this patient's contractions. Terbutaline is a tocolytic. The FDA has indicated that terbutaline should not be used secondary to its side effects and lack of efficacy.

A 33-year-old G2P1 presents at 34 weeks gestation for consultation because ultrasound revealed a 3900 gm fetus with biometrics consistent with 39 weeks. Her prior pregnancy was complicated by gestational diabetes and a shoulder dystocia. Which of the following complications is this fetus at greatest risk? A. Birth trauma B. Hyperglycemia C. Hypobilirubinemia D. Hypothyroidism E. Congenital anomalies

The fetus with enhanced general growth or macrosomia is defined by a birth weight at or above the 90th percentile for gestational age. The condition can usually be ascribed to one of three etiologies: enhanced growth potential (50-60%); abnormal maternal glucose homeostasis (35-40%); or underestimation of fetal age (5%). Macrosomic newborns of diabetic mothers experience excessive rates of neonatal morbidity, including birth trauma such as shoulder dystocia and brachial plexus injury. These infants have significantly higher rates of severe hypoglycemia and neonatal jaundice. Neonatal acidosis occurs with poor glycemic control, thus increasing the incidence of fetal demise. While poorly controlled pre-existing diabetes is associated with an increased risk of congenital anomalies, gestational diabetes is not associated with increased risk of congenital anomalies.

A 42-year-old G5P2 woman at 36 weeks gestation is diagnosed with preeclampsia. Her previous pregnancy was complicated by twins and preeclampsia at 36 weeks gestation. She also has had two spontaneous abortions at seven weeks gestation. Which of the following conditions is not associated with her increased risk for preeclampsia in this pregnancy? A. Previous history of preeclampsia B. Chronic hypertension C. Multifetal pregnancy D. Age E. Previous spontaneous abortion

E. The previous history of spontaneous abortion does not put the patient at increased risk. The incidence of preeclampsia is commonly cited to be about 5 percent and is markedly influenced by parity. It is related to race, ethnicity and genetic predisposition. Environmental factors are also likely to play a role. Other risk factors for preeclampsia include a previous history of the disease, chronic hypertension, multifetal pregnancy and molar pregnancy. In addition, patients at extremes of maternal age or with diabetes, chronic renal disease, antiphospholipid antibody syndrome, vascular or connective tissue disease or triploidy are at increased risk for developing preeclampsia

A 19-year-old G2P1 African American woman at 30 weeks gestation presents with preterm rupture of membranes six hours ago. Her prenatal course has been complicated by two episodes of bacterial vaginosis for which she was treated. She takes prenatal vitamins and iron. She denies substance abuse or alcohol use, but admits to smoking five cigarettes each day. Her prior pregnancy was delivered vaginally at 41 weeks after spontaneous rupture of membranes. Her blood pressure is 110/70; pulse 84; temperature 98.6°F (37.0°C). Pertinent sonographic findings reveal oligohydramnios and a cervical length of 30 mm. Which of the following is the most likely cause of preterm premature rupture of membranes in this patient? A. Ethnicity B. Smoking C. Previous premature rupture of membranes D. Cervical length E. Genital tract infections

E. The primary risk factor for preterm rupture of membranes is genital tract infection, especially associated with bacterial vaginosis. All of the other listed options are risk factors. Smoking and prior preterm premature rupture of membranes (which she did not have previously because she delivered at 41 weeks) increases the likelihood of preterm rupture of membranes two-fold. A shortened cervical length is also a risk factor, but her cervical length is normal.

A 20-year-old G0 woman presents for prenatal care at 10 weeks gestation. She states that she is healthy and denies vaginally bleeding. Prenatal labs reveal a hemoglobin of 9.1 g/dL with the following indices: MCV (mean cellular volume): 72 femtoliter MCH (mean cellular hemoglobin): 22 picograms/cell Serum ferritin: 108 mcq/L Hemoglobin A2: normal Hemoglobin F: normal Hemoglobin electrophoresis: normal What is the most likely cause of anemia in this patient? A. Acute blood loss B. Iron deficiency anemia C. Hemoglobin H disease D. Beta thalassemia E. Alpha thalassemia trait

E. This patient has alpha thalassemia trait characterized by mild anemia, macrocytic and hypochromic anemia and a normal hemoglobin electrophoresis. She denies blood loss therefore acute blood loss is unlikely and her serum ferritin is normal ruling out iron deficiency anemia. Hemoglobin H disease and beta thalassemia are characterized by moderate to severe anemia. Beta-thalassemia would have hemoglobin F as well as hemoglobin A2 on hemoglobin electrophoresis.

A 23-year-old G1P1 woman diagnosed with postpartum depression at three months after a spontaneous vaginal delivery has suicidal ideation and is desperate for help. Which of the following is the most appropriate next step in the treatment of this patient? A. Behavioral psychotherapy B. Anti-depressant medication C. Anti-psychotic medication D. Electroconvulsive therapy (ECT) E. Inpatient psychiatric admission

E. This patient offered thoughts of suicidal ideation, thus inpatient management is the most appropriate choice. While behavioral psychotherapy is necessary to establish long-term strategies for coping skills, newer regimens for postpartum depression include the use of SSRI medication. SSRI medications have been shown to hasten recovery to a fully functioning state. Antipsychotic medication is not indicated without an established diagnosis. Electroconvulsive therapy may be indicated for patients who don't respond to standard depression treatments.

A 38-year-old G2P0 woman at 28 weeks gestation has been diagnosed with preterm labor and is currently stable on nifedipine. Her cervical exam has remained unchanged at 2 cm dilated, 75% effaced and -2 station. Her vital signs are stable and fetal heart tracing is Category I. You recommend treatment with betamethasone (a steroid). Which of the following is associated with betamethasone therapy in the newborn? A. Enhancement of fetal growth B. Increased risk of infection C. Increased incidence of necrotizing enterocolitis D. Increased incidence of intracerebral hemorrhage E. Decreased incidence of intracerebral hemorrhage

E. Treatment with betamethasone from 24 to 34 weeks gestation has been shown to increase pulmonary maturity and reduce the incidence and severity of RDS (respiratory distress syndrome) in the newborn. It is also associated with decreased intracerebral hemorrhage and necrotizing enterocolitis in the newborn. It has not been associated with increased infection or enhanced growth.

A 27-year-old G1P0 woman at 34 weeks gestation presents with increased swelling in her face and hands over the last two days. Blood pressure is 155/99. A 24-hour urine sample for protein is 440 mg/dL. BMI is 27. Repeat blood pressure two days later is 150/92. Which of the following is the most likely diagnosis in this patient? A. Preeclampsia B. Preeclampsia with severe features C. Eclampsia Syndrome D. Gestational hypertension E. Chronic hypertension

A. This patient has met criteria for the diagnosis of mild preeclampsia based on her persistent elevation of blood pressure and 24-hour urine results. The amount of protein excreted in the urine varies throughout the day, therefore a sample is collected over a 24-hour time period. Twenty-four hour urine protein values greater that 300 mg are required for the diagnosis of mild preeclampsia. Values greater than 5000 mg (or 5 g) are required for the diagnosis of severe preeclampsia (assuming no other defining criteria are present such as SBP >160 or DBP >110). This patient has not had a seizure which is the hallmark of eclampsia syndrome.

A 21-year-old G1P0 woman presents to labor and delivery at 39 weeks gestation with a chief complaint of decreased fetal movement over the last two days. An ultrasound shows a fetus with biometry consistent with 34 weeks gestation with no cardiac activity. The head circumference and biparietal diameter are consistent with 37 weeks and the abdominal circumference, femur and humerus lengths are all lagging by approximately five weeks. The amniotic fluid volume is slightly decreased. No other abnormalities are identified. The patient's medical history is notable for a deep venous thrombosis which she had three years ago while she was using oral contraceptives. She had a reassuring quad screen. She denies any history of fever or viral illnesses during the pregnancy. She works as a preschool teacher. The patient had a fetal ultrasound at 20 weeks gestation. At that time all of the fetal anatomy was well-visualized and no abnormalities were identified. Which of the following is the most likely explanation for the fetal demise in this case? A. Trisomy 21 B. Trisomy 18 C. Poorly controlled undiagnosed diabetes mellitus D. Fetal parovirus infection E. Factor V Leiden mutation

A. This patient is most likely to have the autosomal dominant Factor V Leiden (FVL) mutation based on her history. FVL is the most common inherited thrombophilic disorder affecting approximately 5% of Caucasian women in the United States. It is a point mutation which alters factor V making it resistant to inactivation by protein C. The thrombophilic effect of a FVL mutation has been clearly established. Heterozygosity for FVL is associated with a five- to ten-fold increased risk of thrombosis, while homozygosity is associated with an 80-fold increased risk. The FVL mutation is associated with obstetric complications including stillbirth, preeclampsia, placental abruption and IUGR. Fetuses with Trisomy 18 are likely to have congenital anomalies that are detectable on prenatal ultrasound. Over 90% of cases of trisomy 21 and 18 may be detected with the quad screen. A congenital parvovirus infection associated with a fetal demise would likely cause hydrops in the fetus which would be identified on ultrasound. Although poorly controlled diabetes mellitus are associated with fetal demise, they are not the most likely etiologies in this patient whose presentation is classic for the FVL mutation.

A 19-year-old G1P0 woman at 39 weeks gestation presents in labor. She denies ruptured membranes. Her prenatal course was uncomplicated and ultrasound at 18 weeks revealed no fetal abnormalities. Her vital signs are: blood pressure 120/70; pulse 72; temperature 101.0° F (38.3° C); fundal height 36 cm; and estimated fetal weight of 2900 gm. Cervix is dilated to 4 cm, 100% effaced and at +1 station. She receives 10 mg of morphine intramuscularly for pain and soon after has spontaneous rupture of the membranes. Light meconium-stained fluid was noted and, five minutes later, the fetal heart rate tracing revealed variable decelerations with moderate variability. What is the most likely cause for the variable decelerations? A. Umbilical cord compression B. Meconium C. Maternal fever D. Uteroplacental insufficiency E. Morphine administration

A. Variable decelerations are reflex mediated usually associated with umbilical cord compression as a result of cord wrapped around fetal parts, fetal anomalies or oligohydramnios. The presence of light meconium-stained fluid is not associated with a specific fetal heart rate tracing. Uteroplacental insufficiency is associated with late decelerations. Maternal drugs may cause loss of variability.

A 36-year-old female G1 presents for her prenatal care visit at 35 weeks gestation. She has good dating criteria that were confirmed by a first trimester ultrasound. Her previous medical history is positive for hypertension and type 2 diabetes. You have been following fetal growth with serial ultrasounds. At this visit, ultrasound reveals limited fetal growth over the past three weeks. Biometry is consistent with 32-5/7, EFW 2175 g, <10th percentile. What is the most appropriate next test indicated in the management of this patient? A. Amniotic fluid volume, umbilical artery Doppler systolic: diastolic ratio, non-stress test B. Daily fetal kick counts with follow up ultrasound to reassess fetal growth in one week C. Amniocentesis for fetal lung maturity D. Twice daily fetal kick counts with delivery at 37 weeks gestation E. None, delivery is indicated

A. When a pregnancy is complicated by fetal growth restriction, various fetal physiologic parameters require assessment. In growth-restricted pregnancies, oligohydramnios is frequently found. This finding is presumably due to reduced fetal blood volume, renal blood flow and urinary output. Chronic hypoxia is responsible for diverting blood flow from the kidney to organs that are more critical during fetal life. The significance of the amniotic fluid volume with respect to fetal outcome has been well documented. Ninety percent of patients with oligohydramnios delivered growth restricted infants. These infants experienced a high rate of fetal compromise. The systolic/diastolic (S/D) ratio of the umbilical artery is determined by Doppler ultrasound. An increase in the S/D ratio reflects increased vascular resistance. It is a common finding in IUGR fetuses. A normal S/D ratio indicates fetal well-being. As vascular resistance increases, the S/D ratio increases. With severe resistance, there is absence and ultimately reversal of end-diastolic flow. These findings are associated with an increased rate of perinatal morbidity and mortality, and a higher likelihood of a long-term poor neurologic outcome. Options for antenatal testing include the non-stress test, contraction stress test, and the biophysical profile. Any of these may be used in a growth-restricted fetus as a means of detecting possible or probable fetal asphyxia. While fetal kick counts may be of value, additional fetal testing such as twice weekly NST with AFI and weekly umbilical artery Doppler studies is indicated in monitoring fetuses with IUGR.

A 33-year-old G2P1 woman at 39 weeks gestation presents with painful contractions. Her membranes ruptured two hours prior to presentation. Her pregnancy has been uncomplicated and she has a history of a Cesarean section for breech presentation. She highly desires a vaginal birth. On admission, she is having contractions every four minutes and fetal heart tracing is Category I. On cervical exam, she is 5 cm dilated, 80% effaced, and the fetal vertex is at -1 station. Four hours later, she continues to contract every four minutes with reassuring fetal status and her cervical exam is unchanged. What is the next best step in the management of this patient? A. Place an intrauterine pressure catheter B. Administer vaginal prostaglandin E1 C. Administer vaginal prostaglandin E2 D. Perform a vacuum assisted vaginal delivery E. Perform a Cesarean section

A. While the patient is contracting every four minutes, it is not clear if her contractions are adequate. An intrauterine pressure catheter (IUPC) will help determine if her contractions are adequate and if oxytocin augmentation is appropriate. Prostaglandins are used for cervical ripening and are contraindicated in patients with history of previous Cesarean section. While a vacuum assisted delivery is not contraindicated in a patient with prior history of C-section, it should not be performed in a patient who is not completely dilated. A C-section is not indicated yet because it is unclear if the patient is having adequate strength contractions.

A 29-year-old G1P0 woman at 41 weeks gestation presents in early labor. The prenatal course was uncomplicated. Ultrasound at 21 weeks was consistent with gestational age. Her vitals reveal a blood pressure of 128/76; pulse 74; and she is afebrile. Fundal height is 36 cm with estimated fetal weight of 2700 gm. Cervix is dilated to 1 cm, 50% effaced and the fetal vertex is at -2 station. The nurse calls you to evaluate the fetal tracing. Which statement best describes the tracing seen below? (baseline 140s, variability 5-20, no accels no decels. CTX odd pattern) A. Normal fetal heart rate with good variability and regular contractions B. Fetal tachycardia with good variability and regular contractions C. Normal fetal heart rate with poor variability and regular contractions D. Fetal tachycardia with poor variability and irregular contractions E. Normal fetal heart rate with poor variability and irregular contractions

A

A 25-year-old G2P1 woman presents at 26 weeks gestation with preterm labor. She is currently receiving tocolytic therapy with magnesium sulfate. The patient's nurse is concerned the patient may have magnesium sulfate toxicity. The patient is alert and has no complaints. Her contractions have stopped and her vital signs are stable. Which of the following findings associated with magnesium sulfate treatment would this patient experience before she develops respiratory depression? A. Areflexia B. Hyperreflexia C. Tachycardia D. Hypertension E. Oligouria

A. ! High levels of magnesium sulfate may cause respiratory depression (12-15 mg/dl) or cardiac depression (>15 mg/dl). Prior to developing respiratory depression the patient should have diminished or absent deep tendon reflexes (areflexia). Magnesium sulfate does not cause hypertension.

A 37-year-old G2P1 woman with poorly controlled chronic hypertension presents in labor at term. Her prenatal course was uncomplicated. She delivers a 3500 gram infant spontaneously after oxytocin augmentation of labor. Immediately postpartum, she experiences excessive bleeding. Her blood pressure is 130/90; pulse 84; and she is afebrile. On examination, uterine fundus is firm and the placenta is intact. Which of the following is the most appropriate next step in the management of this patient? A. Exploration for lacerations B. Methylergonovine C. B Lynch suture D. IV push of oxytocin E. Uterine artery embolization

A. After ensuring appropriate backup, establishing intravenous access and stabilizing a patient as needed, the first steps in the management of postpartum hemorrhage are to make sure the uterus is well-contracted, there is no retained placental tissue and to look for lacerations. This patient has a firm fundus, which indicates a contracted uterus. Her placenta is complete, which typically rules out retained placental tissue, so it is important to rule out lacerations, which can lead to hemorrhage. Methylergonovine, prostaglandins and oxytocin are all uterotonics and used to increase uterine contractions and decrease uterine bleeding. Methylergonovine is an ergot alkaloid, which is a potent smooth muscle constrictor. It is also a vasoconstrictive agent and should be withheld from women with hypertension and/or preeclampsia. B Lynch suture is used at time of laparotomy for uterine atony. Oxytocin should not be given as an IV push. Uterine artery embolization can be considered after other sources of bleeding such as lacerations are ruled out.

A 33-year-old G2P1 woman at 29 weeks gestation presents with confirmed preterm premature rupture of membranes. She denies labor. She takes prenatal vitamins and iron. She denies substance abuse, smoking or alcohol use. Her prior pregnancy was delivered vaginally at 41 weeks after spontaneous rupture of membranes. Her blood pressure is 110/70; pulse 84; temperature 98.6°F (37.0°C). Which of the following is the next best step in the management of this patient? A. Ampicillin and erythromycin B. Clindamycin and gentamicin C. Nifedipine D. Terbutaline E. Magnesium sulfate

A. Antibiotic therapy with ampicillin and erythromycin given to patients with preterm premature rupture of the membranes has been found to prolong the latency period by 5-7 days, as well as reduce the incidence of maternal amnionitis and neonatal sepsis. Clindamycin and gentamicin are not indicated for the management of PPROM. Tocolytics may also prolong the pregnancy for various lengths of time, but generally not seven days.

A 28-year-old G1 at approximately 40 weeks gestation presents to triage with mild contractions. You measure her fundal height at 34 cm. You are concerned about intrauterine growth restriction and you want to confirm her dates. In reviewing her records, she reports first feeling fetal movements at 18 weeks gestation. The crown-rump length measurements determined at eight weeks and femur length at 20 weeks are consistent with 40 weeks gestation. Today's assessment reveals biometrics consistent with 34 weeks, amniotic fluid index of 1, and placental calcifications. Which of the following is considered the most reliable method of determining the gestational age in this patient? A. Crown-rump length measurement B. Second trimester ultrasound C. Quickening date D. Third trimester composite biometry E. Placental calcifications

A. In order to accurately confirm gestational age at term, one of the following criteria should be met: Fetal heart tones have been documented for 20 weeks by a non-electronic fetoscope or for 30 weeks by Doppler; it has been 36 weeks since a positive serum or urine HCG pregnancy test was performed by a reliable laboratory; an ultrasound measurement of the crown-rump length, obtained at six to twelve weeks, supports a gestational age of at least 39 weeks; and an ultrasound obtained at 13-20 weeks confirms the gestational age of at least 39 weeks, determined by clinical history and physical examination. The crown-rump length can reliably date a pregnancy within five to seven days.

A 29-year-old G1P0 woman at 42 weeks gestation presents to labor and delivery because of intermittent contractions. She denies ruptured membranes. Her prenatal course was uncomplicated. Her vital signs are: blood pressure 140/96; pulse 72; afebrile; fundal height 32 cm; and estimated fetal weight of 2900 gm. Cervix is closed, 25% effaced, -2 station. The fetal heart rate tracing shows occasional late decelerations. Of the following, what is the next best step in management? A. Maternal left lateral position B. Intrauterine resuscitation with terbutaline Start an amnioinfusion Begin magnesium sulfate Augment labor with oxytocin

A. Initial measures to evaluate and treat fetal hypoperfusion include a change in maternal position to left lateral position which increases perfusion to the uterus, maternal supplemental oxygenation, treatment of maternal hypotension, discontinue oxytocin, consider intrauterine resuscitation with tocolytics and intravenous fluids, fetal acid-base assessment with fetal scalp capillary blood gas or pH measurement. An amnioinfusion may be used to treat patients with variable decelerations. Measures to improve uteroplacental blood flow should be attempted prior to proceeding with Cesarean delivery. Magnesium sulfate is not yet indicated in this patient with one slightly elevated blood pressure. Augmentation of labor may accentuate the late decelerations.

A 29-year-old G1P0 woman at 42 weeks gestation presents in labor. She denies ruptured membranes. Her prenatal course was complicated by chronic hypertension. Her vital signs are: blood pressure 130/80; pulse 72; afebrile; fundal height 38 cm; and estimated fetal weight of 3000 gm. Cervix is dilated to 4 cm, 100% effaced, -1 station, and bulging bag of water. The fetal heart rate tracing reveals five contractions in 10 minutes and repetitive late decelerations. What is the most likely cause of her late decelerations? A. Uteroplacental insufficiency B. Umbilical cord compression C. Uterine hyperstimulation D. Intrauterine growth restriction E. Fetal head compression

A. Late decelerations when viewed as repetitive and/or with decreased variability are an ominous sign. The can be associated with uteroplacental insufficiency as a result of decreased uterine perfusion or placental function, thus leading to fetal hypoxia and acidemia. Common causes include chronic hypertension and postdate pregnancies. Variable decelerations are associated with cord compression. Uterine hyperstimulation may cause prolonged bradycardia. While fetuses with intrauterine growth restriction may have late decelerations, the estimated fetal weight of 3000 g is a normal birth weight for this gestational age. Fetal head compression may be associated with early decelerations.

A 24-year-old G1P0 woman at 32 weeks gestation presents with leaking watery fluid from the vagina. On evaluation, preterm premature rupture of membranes is confirmed. She has occasional Braxton Hicks contractions associated with fetal heart rate accelerations. She does not have vaginal bleeding and vaginal fluid phosphatidylglycerol is absent. Her blood pressure is 110/70; pulse 90; temperature 98.6°F (37.0°C). Fundal height is 30 cm and her fundus is tender. Amniotic fluid index (AFI) is 4. Which of the following findings is an indication for delivery in this patient? A. Tender uterine fundus B. Size less than dates C. Fetal heart rate accelerations D. Amniotic fluid index of less than 5 E. Absence of vaginal fluid phosphatidylglycerol

A. Maternal signs of chorioamnionitis or other evidence of intra-amniotic infection are indications for delivery. This patient has ruptured membranes and a tender fundus, which indicate chorioamnionitis. Labor at 32 weeks would be allowed to progress and prolonged non-reassuring fetal testing would prompt delivery. There are no criteria for amniotic fluid index or degree of oligohydramnios as an indication for delivery. Most authors agree that the achievement of fetal lung maturity (i.e. positive phosphatidylglycerol or 34 weeks gestational age) is the threshold at which the risk of morbidity and mortality of maintaining the pregnancy in utero outweighs the benefits of prolonging the pregnancy.

A 27-year-old G1P1 woman with no significant prior medical history reports three months of low energy, lack of enjoyment with her daily activities, early morning awakening, and trouble concentrating. What is the next best step in the assessment of this patient? A. Risk of suicide B. Willingness to accept medical treatment C. Family history of mental illness D. Good support system E. Current medication profile

A. Most depressed patients who are suicidal are relieved to be asked about it. Although all the items listed are components of a complete history, the most important topic is assessment of suicide risk.

A 26-year-old G2P1 woman at 33 weeks gestation presents in preterm labor. She has a history of a prior preterm birth at 32 weeks gestation. She has insulin dependent diabetes and has a history of myasthenia gravis. She has regular contractions every three minutes and fetal heart tones are reassuring. Cervix is 3 cm dilated and 0 station. Her blood pressure is 140/90. Which of the following is the most appropriate tocolytic agent to use in this patient? A. Nifedipine B. Terbutaline C. Magnesium sulfate D. Indomethacin E. Ritodrine

A. Nifedipine, a calcium channel blocker, is the best option for her as she has contraindications to the other agents listed. Terbutaline and ritodrine are contraindicated in diabetic patients and the FDA made a formal announcement in 2011 warning against using terbutaline to stop preterm labor, stating that terbutaline is both ineffective and dangerous if used for longer than 48 hours. Magnesium sulfate is contraindicated in myasthenia gravis. Indomethacin is contraindicated at 33 weeks due to risk of premature ductus arteriosus closure.

A 32-year-old G2P1 woman is at 41 weeks gestation. Her cervix is long and closed. She does not report contractions and states there is active fetal movement. The patient strongly desires to avoid an induction of labor. She would like to wait until she goes into labor spontaneously. Which of the following management options is optimal at this time? A. Perform a non-stress test (NST) and amniotic fluid index (AFI) twice a week with induction of labor for a nonreactive non-stress test or oligohydramnios B. Patient should perform daily fetal movement counts and proceed with induction for decreased fetal movement C. Perform daily biophysical profiles and deliver if 4 or less D. Immediate induction of labor E. Immediate Cesarean section

A. Optimal management for the patient with an unfavorable cervix at 42 weeks gestation is controversial. Induction of labor in a patient with a reactive tracing and an unfavorable cervix will minimize any risk of antepartum fetal demise; however, the risk of Cesarean section is significantly increased compared to a patient who goes into spontaneous labor. It is reasonable to follow a patient who is 41 weeks with antepartum fetal testing, such as twice weekly NSTs with amniotic fluid assessment. The risk of fetal death is 1-2/1,000 high-risk pregnancies with a reassuring non-stress test, contraction stress test or biophysical profile. The addition of amniotic fluid assessment may improve the predictive value of a reactive NST and reduce the risk of antepartum fetal demise to even lower levels.

A 19-year-old G1P0 woman presents in labor at term. Her prenatal course was uncomplicated. She delivers a 3500 gram infant spontaneously after oxytocin augmentation of labor. Postpartum, she experiences excessive bleeding. Which of the following defines postpartum hemorrhage in this patient? A. Greater than 500 cc B. Greater than 750 cc C. Greater than 1000 cc D. Greater than 1500 cc E. Any amount of bleeding that leads to hypovolemia

A. Postpartum hemorrhage is defined as bleeding in excess of 500 cc after a vaginal delivery or in excess of 1000 cc after a Cesarean delivery.

A 29-year-old G1P0 woman is at 42 weeks gestation based on her last menstrual period and a first trimester ultrasound. Of the following, what factor is most likely to be associated with postterm pregnancy? A. Placental sulfatase deficiency B. Fetal adrenal hyperplasia C. Fetal alpha-fetoprotein deficiency D. Fetal renal anomalies E. Fetal chromosomal abnormalities

A. Postterm pregnancies are associated with placental sulfatase deficiency, fetal adrenal hypoplasia, anencephaly, inaccurate or unknown dates and extrauterine pregnancy. Postterm pregnancies are not associated with fetal adrenal hyperplasia, alpha-fetoprotein deficiency, renal anomalies or chromosomal abnormalities.

A 25-year-old G2P1 woman at 20 weeks gestation is diagnosed with preterm premature rupture of the membranes. She denies labor. She takes prenatal vitamins and iron. She denies substance abuse, smoking or alcohol use. Her prior pregnancy was delivered vaginally at 36 weeks after preterm rupture of membranes. Her blood pressure is 110/70; pulse 84; temperature 98.6°F (37.0°C). Her amniotic fluid index is 2. The patient's fetus is greatest risk for which of the following? A. Pulmonary hypoplasia B. Cardiac anamolies C. Urinary tract anamolies D. Microcephaly E. Compression fractures

A. Preterm premature rupture of the membranes that occurs before viability is associated with significant risk of poor outcome. Neonatal survival when rupture occurs between 20 and 23 weeks is approximately 25%. Complications that may be found in the developing fetus include structural abnormalities that are primarily deformations (abnormalities that occur due to an insult after a structure has already formed) rather than malformations (abnormal development of the structure itself). Pulmonary hypoplasia is seen when rupture of membranes occurs before 25 weeks gestation because the lack of amniotic fluid interferes with the normal intrauterine breathing process. The result is failure of normal development and growth of the respiratory tree.

A 32-year-old G1 presents at 35 weeks gestation with decreased fetal movement. Her prenatal course has been complicated by size less than dates. Serial ultrasounds show a decrease of the estimated fetal weight from 60th to 20th percentile. The non-stress test is reactive and the amniotic fluid index is 10. What is your next step in management? A. Continue with weekly non-stress tests B. Obtain umbilical artery systolic: diastolic ratio C. Admission for daily fetal surveillance D. Induction of labor today E. Cesarean section today

A. Since the patient reported decreased fetal movement, a non-stress was performed and was reassuring. The NST is based on the principle that when the fetus moves, its heartbeat normally accelerates. The NST assesses fetal health through monitoring accelerations of the heart rate in response to the baby's own movements. Amniotic fluid volume is important because a decreased amount raises the possibility that the fetus may be under stress. Since the fetus does not show growth restriction and fetal status was reassuring, there are no indications for Doppler studies or delivery. In light of the dramatic decrease in growth, it is reasonable to follow this patient with weekly non-stress tests.

A 24-year-old G1P1 woman just delivered a healthy infant at term. She has a history of a psychiatric disorder and was treated for depression while in college. Which of the following is she at most increased risk for in the postpartum period? A. Postpartum blues B. Postpartum depression C. Postpartum psychosis D. Postpartum anxiety E. She is not at increased risk for a psychiatric disorder

B. A patient's history of a psychiatric illness is a risk factor for the development of a postpartum depression. Patients with a prior history of depression, either situational or spontaneous, are at very high risk for postpartum depression. In fact, one-third of patients with a postpartum psychiatric problem report a prior history. These patients need careful follow up after delivery, which should include an early appointment for a postpartum visit. Questions at this time should be directed to her moods and thoughts.

Thirty-six hours ago a 23-year-old G1P1 woman delivered vaginally and sustained a 2nd-degree laceration. She had a prolonged first stage of labor, ruptured membranes for 26 hours and received penicillin for group B Strep prophylaxis. She now complains of increasing abdominal pain, cramping and heavy, foul-smelling lochia. Her vital signs reveal a temperature of 100.0° F (37.8° C); pulse 80; blood pressure 120/60; and respirations 18. She has a tender uterine fundus that measures at the umbilicus. Her extremities reveal mild bilateral edema; no erythema or tenderness. Blood work reveals a white count of 12.2; hematocrit of 34%; and normal chemistries. Her urinalysis is positive for blood and negative for WBCs, leukocyte esterase and nitrites. In addition to ampicillin, which of the following would be the best antibiotic choice? A. Erythromycin B. Gentamicin C. Doxycycline D. Vancomycin E. Ciprofloxacin

B. Endomyometritis is a common complication of prolonged labor, prolonged rupture of membranes and multiple vaginal examinations. The infection is polymicrobial, mostly anaerobic and requires broad-spectrum antibiotics for treatment until the patient is afebrile for 24 hours. By adding Gentamicin, you are covering the spectrum of gram-negative organisms. Erythromycin provides good coverage for upper respiratory infections. Vancomycin provides good coverage for S. aureus and penicillin-resistant gram-positive bacteria. Ciprofloxacin provides excellent coverage for gram-negative pathogens, including Pseudomonas.

A 37-year-old G3P0 woman at 29 weeks gestation presents with uterine contractions every five minutes. Her cervix is 1 cm dilated and 50% effaced. Fetal fibronectin test is negative. The patient stops having contractions after bedrest and hydration. What is the strength of using a fetal fibronectin test in patients with preterm contractions? A. Positive predictive value B. Negative predictive value C. High sensitivity D. Low false positive rate E. High false positive rate

B. Fibronectin is an extracellular matrix protein that is thought to act as an adhesive between the fetal membranes and underlying decidua. It is normally found in cervical secretions in the first half of pregnancy. Its presence in the cervical mucus between 22 and 34 weeks is thought to indicate a disruption or injury to the maternal-fetal interface. Fetal fibronectin is FDA approved for use in women with symptoms of preterm labor from 24 to 35 weeks and during routine screening of asymptomatic patients from 22 to 30 weeks gestation. Fetal fibronectin has a negative predictive value of 99.2% in symptomatic women — 99 out of every 100 patients with a single negative test result will not deliver in the next 14 days. The positive predictive value in symptomatic women is 16.7% — 17 out of 100 women with a positive test will deliver within 14 days. In asymptomatic women, a negative fetal fibronectin test has a negative predictive value of 96.7% for delivery before 35 weeks.

A 32-year-old G2P1 woman at 36 weeks gestation presents with preterm premature rupture of the membranes that occurred 36 hours ago. She denies labor. She takes prenatal vitamins and iron. She denies substance abuse, smoking or alcohol use. Her prior pregnancy delivered vaginally at 34 weeks after spontaneous rupture of membranes. Her blood pressure is 110/70; pulse 84; temperature 98.6°F (37.0°C). The estimated fetal weight is 2700 grams. She is having one contraction per hour and fetal heart tracing is Category I. Which of the following is the most appropriate next step in the management of this patient? A. Observation until spontaneous onset of labor B. Augmentation of labor C. Magnesium sulfate D. Nifedipine E. Corticosteroids

B. In this patient, the benefits for delivery outweigh the risk of expectant management, so the patient should undergo augmentation of labor. Expectant management at 36 weeks poses a large risk to the development of chorioamnionitis. The role of tocolytics in the setting of preterm premature rupture of membranes is controversial and is contraindicated at 36 weeks gestation. Steroid administration after 32 weeks is controversial.

A 19-year-old G2P1 woman at 28 weeks gestation has been diagnosed with preterm labor. Her physician has chosen to treat her with magnesium sulfate. By what mechanism of action does magnesium sulfate work as a tocolytic? A. Decreases prostaglandin (PG) production B. Competes with calcium for entry into cells C. Increases cAMP in the cell D. Blocks calcium entry into muscle cells E. Inhibits calcium transport

B. Magnesium sulfate works by competing with calcium entry into cells. Beta-adrenergic agents work by increasing cAMP in the cell, thereby decreasing free calcium. Prostaglandin synthetase inhibitors, such as Indomethacin, work by decreasing prostaglandin (PG) production by blocking conversion of free arachidonic acid to PG. Calcium channel blockers prevent calcium entry into muscle cells by inhibiting calcium transport.

A 32-year old G1P0 woman at 29 weeks gestation presents with preterm labor. She is started on indomethacin. What is a possible adverse fetal effect associated with indomethacin treatment? A. Polyhydramnios B. Premature constriction of the ductus arteriosus C. Fetal growth restriction D. Hypoxia E. Chorioamnionitis

B. Maternal indomethacin exposure can result in premature constriction of the ductus arteriosus, especially if used after 32 weeks gestation. Polyhydramnios is not associated with indomethacin. In fact, indomethacin is associated with oligohydramnios. Fetal hypoxia and decreased uteroplacental blood flow have been associated with the use of calcium channel blockers, such as Nifedipine. Indomethacin should not cause chorioamnionitis.

A 36-year-old G1P0 woman presents in active labor. Her past medical history and prenatal course were complicated by chronic hypertension and superimposed preeclampsia. She received magnesium sulfate for seizure prophylaxis and oxytocin augmentation. She undergoes an uneventful spontaneous vaginal delivery. Postpartum, she has a 1000 ml hemorrhage due to uterine atony. Her blood pressure is 130/80; pulse 96; and she is afebrile. Which of the following uterotonic agents is contraindicated in this patient? A. Oxytocin B. Methylergonovine C. Prostaglandin F2-alpha D. Prostaglandin E2 E. Misoprostol

B. Methergine, prostaglandins, misoprostol, and oxytocin are all uterotonics and used to increase uterine contractions and decrease uterine bleeding. Methylergonovine is an ergot alkaloid, which is a potent smooth muscle constrictor. It is also a vasoconstrictive agent and should be withheld from women with hypertension and/or preeclampsia. Misoprostol, prostaglandin E1, used for cervical ripening and labor induction, is a uterotonic agent frequently used for uterine atony, although not FDA approved for this use.

A 29-year-old G1P0 woman at 31 weeks gestation presents with watery discharge from the vagina commencing several hours ago. Her prenatal course has been uncomplicated and she takes prenatal vitamins and iron. She denies substance abuse, smoking or alcohol use. On examination, her blood pressure is 110/70; pulse 84; temperature 98.6°F (37.0°C). Which of the following is the most appropriate next step in the management of this patient? A. Nitrazine testing of mucus swabbed from cervix B. Examination of vaginal fluid for ferning C. Digital examination of cervix D. Determination of amniotic fluid index (AFI) E. Non-stress test

B. Methods to confirm rupture of membranes include testing the vaginal fluid for ferning and nitrazine testing. It is important to test the fluid from the vagina and not to test cervical mucus because of false positive ferning patterns. A digital exam should be avoided in a patient you suspect might have preterm rupture of membranes because of the risk of introducing bacteria into the uterine cavity and increasing risk for chorioamnionitis. Determination of AFI with ultrasound may reveal oligohydramnios and support the diagnosis of rupture of membranes, but does not confirm this diagnosis. Similarly, a non-stress test may reveal variable decelerations, which may be present in the setting of rupture of membranes.

A 29-year-old G1P0 woman at 41 weeks gestation presents for a prenatal visit. Her prenatal course is complicated by tobacco abuse and intermittent prenatal care. Her last visit was at 35 weeks. Prenatal labs are unremarkable except cervical DNA probe positive for Chlamydia, which was treated, and a Pap smear with low-grade squamous intraepithelial lesion. Ultrasound at 21 weeks was consistent with gestational age based on her certain regular LMP. Her vitals reveal a blood pressure of 128/76; pulse 74; and temperature 98° F (36.7° C). Fundal height is 39 cm with estimated fetal weight of 2700 gm. Cervix is dilated to 1 cm, 50% effaced, -2 station. What is the next best step in the management of this patient? A. Return visit in one week B. Non-stress test and assessment of amniotic fluid volume C. Ultrasound to assess fetal growth D. Oxytocin challenge test E. Cesarean section

B. Postterm pregnancies should be followed with antepartum fetal surveillance because perinatal morbidity and mortality increases beginning at 41 weeks of gestation. Many practitioners use twice-weekly testing with some evaluation of amniotic fluid volume beginning at 41 weeks of gestation. A non-stress test and amniotic fluid volume assessment (a modified BPP) should be adequate. The non-stress test is an assessment of fetal well-being that measures the fetal heart rate response to fetal movement. The normal or reactive non-stress test occurs when there are two fetal heart rate accelerations of 15 beats/minute for 15 seconds within 20 minutes. Contraction stress test assesses uteroplacental insufficiency and looks for persistent late decelerations after contractions (3/10 minutes); however, it is not necessary to perform, as the non-stress test will assess fetal well-being, as well. An ultrasound to assess fetal growth is not indicated as the patient's fundal height is appropriate for her gestational age and she does not have any other indication to assess fetal growth such as a history of chronic hypertension or diabetes. Observation alone would not be proper care as the patient is postterm. Delivery is indicated if there is evidence of fetal compromise or oligohydramnios.

A 22-year-old G1P1 woman with no prenatal care delivered a 2100 g female infant. The infant had a spontaneous cry upon delivery. The baby's father is concerned because his new daughter's skin appears to be peeling and has a green/yellow hue, and her fingernails are very long. Overall both parents are concerned, as the baby appears to be thin and fragile. Which of the following diagnoses best explains these findings? A. Acute fetal hypoxia B. Fetal dysmaturity C. Trisomy 18 D. Trisomy 21 E. Fragile X syndrome

B. The incidence of infants with dysmaturity approaches 10% when the gestational age exceeds 43 weeks. Infants are described as withered, meconium stained, long-nailed, fragile and have an associated small placenta. These infants are at great risk for stillbirth. The diagnosis of dysmaturity is more common in women with unknown last menstrual periods and unsure dating. While low birth weight is a common finding in infants with Trisomy 18, overlapping fingers, micronathia, and cardiac defects are the most common findings. Trisomy 21 can be associated with low birth weight, but the syndrome is characterized by a constellation of facial findings (low set ears, flattened bridge of the nose, and almond shaped eyes) and nearly 50% are associated with cardiac defects. Acute fetal hypoxia can be associated with meconium stained amniotic fluid but not the other findings noted in this patient. The hallmark of Fragile X syndrome (more common in males than females) is developmental delay not apparent at birth.

A 24-year old G1P1 woman had an uncomplicated Cesarean section 20 hours ago under general anesthesia secondary to an umbilical cord prolapse. You are called to evaluate her because her temperature is 102.0° F (38.9° C). The patient does not have any specific complaints. She has experienced intermittent chills. Her exam is non-focal. There is no uterine tenderness. A urine analysis shows no WBCS and is nitrate and leukocyte estrase negative. What is the next appropriate step in the management of this patient? A. Endometrial cultures B. Chest X-ray C. Treatment with intravenous broad spectrum antibiotics D. Pelvic ultrasound E. Culture of the incision

B. The lungs are the most common source of fever on the first postpartum day, particularly if the patient had general anesthesia. Atelectasis may be associated with a postpartum fever. Aspiration pneumonia should be considered in patients who had general anesthesia. Ideally antibiotic treatment should be targeted at a specific source of infection. Other sources of infection to consider in a febrile postpartum patient include endometritis, mastitis, the urinary tract, the wound, and the extremities (thrombophlebitis). Endometrial and incision cultures are not indicated since her uterus is non-tender and the incision appears normal.

A 28-year-old G1P1 woman delivered four days ago. She tearfully reports that she had trouble sleeping, felt anxious and has been irritable for the last two days. She feels somewhat better today, but is still concerned. What is the most likely diagnosis? A. Hypothyroidism B. Postpartum blues C. Depression D. Normal postpartum state E. Anxiety

B. The patient is describing symptoms of postpartum blues that affects 40-80% women within two to three days postpartum and resolve within two weeks. Symptoms include insomnia, easy crying, depression, poor concentration, irritability or labile affect and anxiety. Symptoms often last a few hours per day and are mild and transient. While hyperthyroidism and anxiety may cause insomnia, this is not the most likely explanation in this patient. Postpartum depression symptoms, such as mood changes, insomnia, phobias and irritability are more pronounced than with the blues and last longer than two weeks.

A 34-year-old G1P0 woman is in a motor vehicle accident. While in the emergency department, the doctors order multiple x-rays to evaluate her injuries. At what gestational age would the fetus be most susceptible to developing intellectual disability with sufficient doses of radiation? A. 0-7 weeks B. 8-15 weeks C. 16-25 weeks D. 26-30 weeks E. 31-35 weeks

B. The risk of developing microcephaly and severe intellectual disability is greatest between eight and 15 weeks gestation. In 1990, the Committee on Biological Effects reported that no risk of mental retardation (now referred to as intellectual disability) has been documented with doses even exceeding 50 rad at less than eight weeks or greater than 25 weeks gestation.

A 19-year-old G1P0 woman at 18 weeks gestation presents with a three-month history of palpitations and intermittent chest pain. Physical examination reveals a pulse of 96 and grade II/VI systolic ejection murmur with a click. The ECG shows normal rate and rhythm and an echocardiogram is ordered. Which of the following is the best treatment in the management of this patient? A. Anxiolytics B. β-blockers C. Calcium-channel blockers D. Digitalis E. No treatment needed at this time

B. This patient has classic symptoms of mitral valve prolapse. Most women with mitral valve prolapse are asymptomatic and diagnosed by routine physical examination or as an incidental finding at echocardiography. A small percentage of women with symptoms have anxiety, palpitations, atypical chest pain, and syncope. For women who are symptomatic, b-blocking drugs are given to decrease sympathetic tone, relieve chest pain and palpitations, and reduce the risk of life-threatening arrhythmias. Because she is symptomatic, the option of no treatment is not correct.

A 23-year-old G1P0 woman at 24 weeks gestation requires treatment for depression. She has no other pregnancy complications. In addition to counseling, she begins therapy with fluoxetine (Prozac). Which of the following symptoms is the most common side effect of her therapy? A. Fatigue B. Sleep disturbance C. Headache D. Irritability E. Agitation

B. While all the side effects listed are reported in patients on fluoxetine, an SSRI antidepressant medication, the most common side effect is insomnia. Significant insomnia may affect one in five patients taking SSRIs. In addition to sleep disturbances, sexual dysfunction, such as decreased libido and delayed or absent orgasm, are common.

A 24-year-old G4P0 woman presents to your office at seven weeks gestation after two days of bleeding and cramping. She thinks that she miscarried at home and brought in the tissue for pathologic evaluation. What is the karyotype most likely to be found on chromosomal analysis? A. Turner Syndrome (45, X) B. Autosomal Trisomy C. Monoploidy D. Triploidy E. Tetraploidy

B. Autosomal trisomy is the most common abnormal karyotype encountered in spontaneous abortuses, accounting for approximately 40-50% of cases. The most common chromosomal aneuploidy noted in abortuses is Trisomy 16. Triploidy accounts for approximately 15%, and tetraploidy for 5% of cases. Monosomy X (45X, 0) is seen in 15-25% of losses.

19-year-old G1 woman at 36 weeks gestation presents for her first prenatal visit, stating she was recently diagnosed with HIV after her former partner tested positive. The HIV Western Blot is positive. The CD4 count is 612 cells/µl. The viral load is 9,873 viral particles per ml of patient serum. Which of the management options would best decrease the risk for perinatal transmission of HIV? A. Treatment with intravenous zidovudine at the time of delivery B. Treatment of the newborn with oral zidovudine only if HIV-positive C. One week maternal treatment with zidovudine now D. Cesarean section in second stage of labor E. Single drug therapy to minimize drug resistance

A. Antiretroviral therapy should be offered to all HIV-infected pregnant women to begin maternal treatment as well as to reduce the risk of perinatal transmission regardless of CD4+ T-cell count or HIV RNA level. The baseline transmission rate of HIV to newborns can be reduced from about 25% to 2% with the HAART (highly active antiretroviral therapy) protocol antepartum and continuing through delivery with intravenous zidovudine in labor and zidovudine treatment for the neonate. Cesarean section prior to labor can reduce this rate to 2% (although the benefit is less clear in women with viral loads).

A 35-year-old woman presents to the emergency department with heavy vaginal bleeding at seven weeks gestation. On examination, she has a dilated cervix with blood and tissue present at the cervical os. Which of the following is the most likely chromosomal abnormality to be found in the karyotypic evaluation of the products of conception? A. Autosomal trisomy B. Triploidy C. Tetraploidy D. Monosomy X (45X,0) E. Fragile X mutation

A. Autosomal trisomy is the most common abnormal karyotype encountered in spontaneous abortuses, accounting for approximately 40-50% of cases. Triploidy accounts for approximately 15%, tetraploidy 5% of cases, and Monosomy X (45X, 0) identified in 15-25% of losses. The Fragile X mutation involves an expanded number of trinucleotide repeats in the CGG (cytosine-guanine) sequence.

A 24-year-old G1P0 woman at 22 weeks gestation with systemic lupus erythematosus (SLE) presents complaining of malaise, joint aches, and fever. Physical examination reveals the following: pulse 88; temperature 98.6°F (37.0°C); respiratory rate 22; and BP 150/110 (baseline is 100/70). Laboratory analysis reveals 1 + proteinuria, AST 35, and ALT 28. Which of the following is the most appropriate initial therapy for the treatment of this patient? A. Steroids B. Nonsteroidal anti-inflammatory drugs (NSAIDs) C. Azathioprine D. Cyclophosphamide E. Magnesium sulfate

A. Lupus is notoriously variable in its presentation, course, and outcome. Clinical manifestations include malaise, fever, arthritis, rash, pleuro-pericarditis, photosensitivity, anemia, and cognitive dysfunction. A significant number of patients have renal involvement. There is no cure and complete remissions are rare. Mild disease may be disabling because of pain and fatigue. Nonsteroidal anti-inflammatory drugs are used to treat arthralgia and serositis. Severe disease is best treated with corticosteroids. Hydroxychloroquine is used to help control skin manifestations and may be associated with lupus flares if discontinued. Azathiopine and cyclophosphamide are not indicated in this patient.

A 40-year-old G1P0 woman at 34 3/7 weeks gestation was found on the floor at work having a grand mal seizure. Her airway was secured. Blood pressure in the ambulance was 140/90. What is the initial treatment for her condition? A. Magnesium sulfate B. Valium C. Hydantoin D. Phenobarbital E. Naloxone

A. Magnesium sulfate is the treatment of choice for eclampsia, which is her most likely diagnosis. Valium, hydantoin, tiagabine, and barbiturates can also be used to treat seizures, but are not first-line therapy for eclampsia. They can be added as second agents, or used if magnesium is contraindicated. Naloxone (Narcan) is a drug used to counter the effects of opioid overdose, for example heroin or morphine overdose, and is specifically used to counteract life-threatening depression of the central nervous system.

A 26-year-old G2P0 woman presents for counseling following manual vacuum aspiration of an eight-week missed abortion. The patient asks whether an uncomplicated first trimester pregnancy termination three years ago might have predisposed her to the subsequent spontaneous abortion. What are the patient's risks associated with the prior surgical abortion in the first trimester? A. Does not predispose the patient to subsequent spontaneous abortion B. Increases the risk of spontaneous abortion two-fold C. Predisposes the patient to primary infertility D. Increases the likelihood of subsequent pregnancy loss in both the first and second trimesters E. Increases the likelihood of spontaneous abortion and future delivery complications

A. Neither controlled trials nor surveillance data support the contention that a single, prior first trimester surgical abortion increases the risk of subsequent first trimester pregnancy loss. Indeed, first trimester surgical abortion confers no subsequent obstetric disadvantage, particularly when compared with an appropriate control population. The clinician should reassure this patient that first trimester spontaneous abortion is a common occurrence and that she has not caused this missed abortion.

A 24-year-old G1P0 woman at 12 weeks gestation presents for prenatal care. She is 5 feet 4 inches tall and weighs 220 pounds (BMI: 37.8 kg/m2). She wants to know if there is an increased risk on her pregnancy because of her size. Which of the following is the most common complication in this patient? A. Hypertension B. Preterm labor C. Post-term pregnancy D. Small for gestational age E. Shoulder dystocia

A. The body mass index (BMI) is equal to a person's weight in kg divided by their height in meters squared. The National Heart, Lung, and Blood Institute identify a normal BMI as 18.5 to 24.9 kg/m2; overweight as a BMI of 25 to 29.9 kg/m2; and obesity as a BMI of 30 kg/m2 or greater. Obesity is further categorized as class I (BMI: 30 to 34.9 kg/m2), class II (BMI: 35 to 39.9 kg/m2), and class III (BMI: > or = 40 kg/m2). Increased maternal morbidity results from obesity and includes chronic hypertension, gestational diabetes, preeclampsia, fetal macrosomia, as well as higher rates of Cesarean delivery and postpartum complications. This patient's BMI is approximately 38 so she is a class II and has over a 7-fold increase risk for preeclampsia and a 3-fold risk for hypertension.

A 23-year-old G1P0 woman presents with cramping, vaginal bleeding and right lower quadrant pain. Her last normal menstrual period occurred seven weeks ago. On physical exam, vital signs are: blood pressure 110/74; pulse 82; respirations 18; and temperature 99.4°F (37.4°C). On abdominal exam, she has very mild right lower quadrant tenderness. On pelvic exam, she has scant old blood in the vagina and a normal appearing cervix. Her uterus is normal size and slightly tender. On bimanual exam, there is no cervical motion tenderness, and she has slight tenderness in right lower quadrant. Quantitative Beta-hCG is 2500 mIU/ml; progesterone 6.2 ng/ml; hematocrit 34%. The transvaginal ultrasound shows an empty uterus with endometrial thickening, a mass in the right ovary measuring 3 x 2 cm and a small amount of free fluid in the pelvis. Which of the following is the most appropriate next step in the management of this patient? A. Methotrexate B. Antibiotics C. Observation D. Dilation and curettage E. Culdocentesis

A. The next best step in management is methotrexate administration. Certain conditions must be met prior to initiating methotrexate therapy for treatment of an ectopic pregnancy. These include: hemodynamic stability; non-ruptured ectopic pregnancy; size of ectopic mass <4 cm without a fetal heart rate or <3.5 cm in the presence of a fetal heart rate; normal liver enzymes and renal function; normal white cell count; and the ability of the patient to follow up rapidly (reliable transportation, etc.) if her condition changes. There is no indication for antibiotics in this scenario. Offering observation delays treatment and pain control would not address the underlying cause of the patient's problem. Culdocentesis is not indicated and would not change the management of this patient.

A 28-year-old G0 woman presents to your office for preconception counseling. She has a history of type 1 diabetes, diagnosed at age six, and uses an insulin pump for glycemic control. She has a history of proliferative retinopathy treated with laser. Her last ophthalmologic examination was three months ago. Her last hemoglobin A1C (glycosylated hemoglobin level) six months ago was 9.2%. Which of the following complications is of most concern for her planned pregnancy? A. Fetal growth restriction B. Fetal cardiac arrhythmia C. Twins D. Oligohydramnios E. Macrosomia

A. The patient with type 1 diabetes is at risk for many pregnancy complications. In women with insulin-dependent diabetes, the rates of spontaneous abortion and major congenital malformations are both increased. The risk appears related to the degree of metabolic control in the first trimester. Overt diabetic patients are also at an increased risk for fetal growth restriction, although fetal macrosomia may also occur. The former becomes a greater concern as in this patient, with longer-term diabetes and vascular complications, such as retinopathy. Diabetics also have increased risk for polyhydramnios, congenital malformations (cardiovascular, neural tube defects, and caudal regression syndrome), preterm birth and hypertensive complications. Her diabetes does place her at an increased for twins.

A 24-year-old Rh-negative G1P1 woman just delivered a healthy term infant who is Rh-positive. You recommend RhoGAM administration but she declines because she does not desire any blood products. What is her approximate risk of isoimmunization if she does not receive the RhoGAM? A. Less than 20% B. 40% C. 60% D. 80% E. 100%

A. The risk of isoimmunization is 2% antepartum, 7% after full term delivery, and 7% with subsequent pregnancy so less than 20% total. While 75% of all gravidas have evidence of transplacental hemorrhage during pregnancy or immediately after delivery, 60% of these patients have <0.1 cc of fetal blood in the maternal circulation, which is enough to sensitize a patient. The incidence and size of transplacental hemorrhage increases as pregnancy advances. During the second month of gestation, 5-15% of women will have evidence of feto-maternal hemorrhage. By the third trimester, this number increases to 45% of patients.

A 27-year-old G1P0 woman at 14 weeks gestation presents with a two-month history of insomnia, feeling depressed, and unintentional weight loss. Symptoms began after the unexpected death of her father. She is not excited about this pregnancy and reports no suicidal ideation. Physical examination reveals a woman of stated age with a flat affect. Which of the following therapies is contraindicated in this patient? A. Paroxetine (Paxil®) B. Sertraline (Zoloft®) C. Fluoxetine (Prozac®) D. Citalopram (Celexa®) E. Bupropion (Wellbutrin®)

A. This patient has classic depression. The most commonly used antidepressants are the selective serotonin reuptake inhibitors (SSRIs). One SSRI, paroxetine (Paxil) has been changed to a category D drug because of the increased risk of fetal cardiac malformations and persistent pulmonary hypertension. Other SSRI compounds, fluoxetine, sertraline, and citalopram have not been reported to cause early pregnancy loss or birth defects in animals or in humans. Because these agents have few side effects compared with other antidepressants, they are a good choice for pregnant women. Bupropion is not an MAO inhibitor, nor is it an SSRI and a report by the Bupropion Pregnancy Registry reports no unusual effects in 90 exposed pregnancies.

A 34-year-old G4P3 woman at 19 weeks gestation presents to the emergency department with chest pain, palpitations and sweating, which began 2-3 hours ago. On further questioning, she states that she has been very anxious lately and is not sleeping well, which she attributes to the pregnancy. She reports that she has lost 40 pounds in the last year without trying. She denies significant medical problems. On exam, the patient appears diaphoretic and anxious, her eyes are wide open, prominent, with easily visible sclera surrounding the pupil. Vital signs are: temperature 100.2°F (37.9°C); pulse 132; and blood pressure 162/84. Her height is 5 feet 10 inches and weight is 128 pounds. Her thyroid is palpably enlarged, with an audible bruit. Electrocardiogram shows sinus tachycardia. Remaining labs are pending. Which of the following therapies is contraindicated at this time? A. Radioactive iodine (I-131) B. Propylthiouracil (PTU) C. Propranolol D. Inorganic iodide E. Intravenous fluid replacement

A. This patient is in thyroid storm, an acute, life-threatening, hypermetabolic state. Radioactive iodine (I-131) concentrates in the fetal thyroid and may cause congenital hypothyroidism, so it should not be intentionally used in pregnancy. Acute treatment of thyroid storm may include thioamides (i.e. PTU), propranolol, sodium iodide and dexamethasone. Oxygen, digitalis, antipyretics and fluid replacement may also be indicated. Maternal mortality with thyroid storm exceeds 25%.

A 27-year-old G1P0 woman presents to labor and delivery and is found to have a fetal demise at 34 weeks gestation. She did not have access to prenatal care during the pregnancy. Her vital signs are normal and she is not in labor. Her uterus is non-tender and she does not have any vaginal bleeding or ruptured membranes on exam. Which untreated condition is the most likely cause? A. Diabetes B. Anemia C. Hypothyroidism D. Herpes E. Rh-isoimmunization

A. Uncontrolled glucose is associated with adverse fetal outcome. A patient with type 1 diabetes is at risk for many pregnancy complications, including fetal death and fetal macrosomia, although fetal growth restriction may also occur. Diabetics also have increased risk for polyhydramnios, congenital malformations (cardiovascular, neural tube defects, and caudal regression syndrome), preterm birth, and hypertensive complications. The anemia most often seen in pregnancy is mild and would not be the most likely cause. It is unlikely that she has an abruption causing the anemia, but this should be considered. Hypothyroidism is usually associated with menstrual irregularities and infertility, and is a less likely cause. Rh sensitization is unlikely since this is her first pregnancy and she did not have any bleeding or procedures during the pregnancy.

A 26-year-old G1P0 woman with last menstrual period 13 weeks ago presents to your office for her first prenatal visit. She reports vaginal spotting for the last two days. You perform an ultrasound that shows an intrauterine pregnancy consistent with 11 weeks gestation with no cardiac activity. She denies cramping or abdominal pain. What is the most important laboratory test to check for this patient? A. Quantitative Beta-hCG B. Maternal blood type C. Hemoglobin and hematocrit D. Platelet count E. Progesterone

B. A maternal blood type should be checked on all women with vaginal bleeding during pregnancy, unless it was documented earlier in the pregnancy. If the patient's blood type is Rh-negative, RhoGAM would be indicated to prevent Rh sensitization. Serial quantitative Beta-hCGs can be useful in confirming an ongoing pregnancy before fetal heart rate activity can be noted, but in this case there is a non-viable intrauterine pregnancy. The patient is unlikely to have had significant blood loss making a blood or platelet count unlikely to be necessary at this time. Although progesterone levels can be useful in determining if a pregnancy is failing, the diagnosis is already clear in this case.

A 25-year-old G0 woman presents to the clinic for follow-up after having a first trimester spontaneous abortion. She wants to discuss the cause of this event. Which of the following etiologic categories accounts for the majority of first trimester spontaneous abortions? A. Immunologic abnormalities B. Conceptus genetic anomalies C. Maternal genetic anomalies D. Structural/uterine anomalies E. Uterine infections

B. Although investigators have implicated all of the listed categories as possible causes of spontaneous abortion, genetic abnormalities involving the conceptus account for the majority. In fact, approximately 50 to 60 percent of embryos and early fetuses that are spontaneously aborted contain form of chromosomal abnormalities.

A 34-year-old G1P0 woman is diagnosed with a fetal demise at 37 weeks. She and her husband seem stunned. They cannot believe the news. What is the next psychological response you would expect from this couple after the initial denial? A. Depression B. Anger C. Assessment D. Bargaining E. Acceptance

B. Couples who are presented with the news of a fetal birth defect or loss progress through a series of coping responses. The response to "bad news" varies with the severity, treatability and the coping level of the couple. As an individual starts to understand the situation, frustration or anger may be self-directed or directed to the spouse, the affected child or the caregiver without a rational basis. This is important to recognize to help the couple through these stages: Denial, Anger, Bargaining, Depression, Acceptance.

A 33-year-old G2P1 woman returns at 16 weeks gestation for a follow-up prenatal visit. Her first pregnancy was uncomplicated and delivered at term. Fundal height is 22 cm. Maternal serum alpha-fetoprotein is 3.0 MoMs (multiples of the median). She has not felt fetal movement. What is the next best step in the management of this pregnancy? A. Repeat the maternal serum AFP B. Fetal sonogram C. Biophysical profile D. Fetal Doppler studies E. Amniocentesis

B. In pregnancies with size greater than dates and an elevated maternal serum AFP, you should consider multiple gestation as the etiology. Repeating the maternal serum AFP would only delay further workup. A biophysical profile or fetal Doppler studies are not indicated or performed at this gestational age. Amniocentesis is invasive and would not be utilized prior to performing a fetal survey by ultrasound.

A 38-year-old G5P4 woman with a history of four Cesarean deliveries is at 36 weeks gestation with a singleton pregnancy. She presents to labor and delivery with complaints of vaginal bleeding for the last hour. Prenatal care has been unremarkable except for a second trimester ultrasound discovering an anterior placenta, which partially covers the cervical os. Follow up ultrasound exams have confirmed these findings. The patient denies uterine contractions and abdominal pain. She feels the baby moving. Her blood pressure is 110/60; pulse 110; and she is afebrile. Her abdomen and uterus are non-tender and soft. Fetal heart tones have a baseline of 140 and are reassuring. This patient is at greatest risk for which of the following complications? A. Vasa previa B. Placenta accreta C. Placental abruption D. Uterine rupture E. Preterm labor

B. Placenta accreta occurs when the placenta grows into the myometrium. This patient is at risk for this condition due to her history of four previous Cesarean deliveries, and the low anterior placenta. The scar tissue from the previous surgery prevents proper implantation of the placenta and it subsequently grows into the muscle. Vasa previa is a rare condition where the umbilical cord inserts into the membranes. Placental abruption is the premature separation of the normally implanted placenta. Risk of uterine rupture could be as high as 5% in this case, and the risk of placenta accreta with four prior Cesarean deliveries approaches 50%. The patient is not experiencing contractions at the present time, so preterm labor is unlikely.

A 22-year-old G1P0 woman at 39-weeks gestation presents in active labor. Her pregnancy is complicated by diet-controlled gestational diabetes. She has a history of uterine fibroids. On examination, she is found to be 4 cm dilated in breech presentation. An ultrasound confirms the breech presentation, amniotic fluid index is 5, and the estimated fetal weight is 3900 g. Which of the following is the most likely cause of the breech presentation in this patient? A. Gestational diabetes B. Uterine fibroids C. Oligohydramnios D. Macrosomia E. Gravidity

B. Prematurity, multiple gestation, genetic disorders, polyhydramnios, hydrocephaly, anencephaly, placenta previa, uterine anomalies and uterine fibroids are all associated with breech presentation.

A 28-year-old Rh negative G1P0 woman at eight weeks gestation presents to the clinic for a first prenatal visit. Which of the following is the current recommendation for RhoGAM administration to prevent Rh isoimmunization? A. Routine administration for every Rh-sensitized woman at term B. Administration for Rh-negative patients with no Rh antibodies at 28 weeks C. Administration for every Rh-negative woman who delivers an Rh-negative infant D. Routine administration for all Rh-negative patients during first trimester E. Routine administration for all Rh-negative patients during each trimester

B. RhoGAM (Anti-D-immunoglobulin) is administered to Rh-negative women to prevent isoimmunization. Each dose provides 300 micrograms of D-antibody and is given to the D-negative non-sensitized mother to prevent sensitization after any pregnancy-related events that could result in fetal-maternal hemorrhage. Up to 2 percent of women with a spontaneous abortion and 5 percent of those undergoing elective termination may become isoimmunized without D-immunoglobulin. The current recommendations for Rh-negative women without evidence of Rh immunization is prophylactically at 28-weeks gestation (after an indirect Coombs' test), and within 72 hours of delivering an Rh-positive baby, following spontaneous or induced abortion, following antepartum hemorrhage and following amniocentesis or chorionic villus sampling. If the father of the fetus is known to be Rh-negative, RhoGAM is not necessary since the fetus will be Rh-negative and not at risk for hemolytic disease.

A 41-year-old G3P2 woman presents with cramping, vaginal bleeding and right lower quadrant pain for five days which has progressively worsened. Her last normal menstrual period occurred seven weeks ago. Her surgical history is notable for a bilateral tubal ligation following her last delivery. On physical exam, vital signs are: blood pressure 110/74; pulse 82; respirations 18; temperature 98.6°F (37.0°C). On abdominal exam, she has right lower quadrant tenderness, with rebound and bilateral guarding in the lower quadrants. On pelvic exam, she has scant old blood in the vagina and a normal appearing cervix. Her uterus is normal size and slightly tender. She has cervical motion tenderness on bimanual examination, and marked tenderness on rectal examination. Her quantitative Beta-hCG is 4000 mIU/ml; progesterone 6.2 ng/ml; hematocrit 34%; and WBC 15,400/mcL, with 88% segmented neutrophils and no bands. The transvaginal ultrasound shows an empty uterus with endometrial thickening, a mass in right ovary measuring 3.8 x 2 cm, and a small amount of free fluid in the pelvis. What is the most likely diagnosis in this patient? A. Pelvic inflammatory disease B. Ectopic pregnancy C. Heterotopic pregnancy D. Missed abortion E. Ruptured corpus luteum cyst

B. The diagnosis of ectopic pregnancy is made when either: 1) a fetal pole is visualized outside the uterus on ultrasound; 2) the patient has a Beta-hCG level over the discriminatory zone (the level at which an intrauterine pregnancy should be seen on ultrasound, usually 2000 mIU/ml) and there is no intrauterine pregnancy (IUP) seen on ultrasound; or 3) the patient has inappropriately rising Beta-hCG level (less than 50% increase in 48 hours) and has levels which do not fall following diagnostic dilation and curettage. This patient meets criteria #2, as her Beta-hCG is >2000 mIU/ml with no intrauterine pregnancy seen on ultrasound. The history, physical exam and lab data are not consistent with pelvic inflammatory disease, ovarian torsion, appendicitis or a ruptured corpus luteum cyst. With a heterotopic pregnancy, there should be a visible pregnancy in the uterus. With a missed abortion there should also be some visible tissue or a fetal pole within the uterus.

A 30-year-old G2P1 woman with last menstrual period 10 weeks ago presents for her first prenatal care visit. She is healthy and takes no medications. Her previous pregnancy was an uncomplicated vaginal delivery at 39 weeks. On examination, her vital signs are normal. Her exam is notable for a uterus measuring 14 weeks gestation. Ultrasound shows a diamniotic monochorionic twin gestation at 10 weeks. Which of the following obstetrical complications is more likely in this pregnancy compared to her previous pregnancy? A. Low maternal weight gain B. Congenital anomalies C. Induction after 40 weeks D. Macrosomia E. Rh isoimmunization

B. The incidence of congenital anomalies is increased in twins, particularly monozygotic twins, compared to singletons. The majority of twin pairs in which an anomaly is present will be discordant for the anomaly. Twin gestations tend to deliver earlier than singleton gestations, with the average length of twin gestation being 35-37 weeks. The optimal length of twin gestation is a matter of some controversy. An observational study comparing perinatal mortality among twin and singleton gestations showed that perinatal mortality reached a nadir at 37-38 weeks in twins and then increased. There have been no prospective studies to demonstrate that induction of labor after 38 weeks in twin gestations improves perinatal outcome. Twins typically weigh less than singletons of the same gestational age, but their weights usually remain within the normal range. Macrosomia is, therefore, uncommon. Rh isoimmunization is not increased in twin gestations.

A 40-year-old G1P0 woman at 22 weeks gestation presents to the office with a complaint of pelvic pressure. She reports that she had intercourse the night prior to presentation and noted some mucous mixed with blood this morning. Her history is significant for type 1 diabetes and she is on an insulin pump. Her surgical history is significant for a history of cone biopsy for treatment of abnormal Pap smear three years ago. On examination, her BMI is 26. She is noted to have a 2 cm dilated cervix with bulging membranes that rupture upon placing the speculum. Fetal parts are noted in the vagina. What is the most likely cause of this finding? A. Uncontrolled diabetes B. Cervical incompetence/insufficiency C. Preterm labor D. Advanced maternal age E. Infection

B. The most likely cause of painless cervical dilation which leads to pelvic pressure, bulging membranes and fetal loss is cervical incompetence or insufficiency. This patient has a history of cone biopsy which can lead to cervical incompetence. Preterm labor by definition does not occur until 24 weeks gestation. Although uncontrolled diabetes can lead to fetal malformations and early miscarriage, it is not typically a cause of fetal loss in the second trimester. Advanced maternal age is associated with an increased risk of stillbirth, preeclampsia, gestational diabetes and intrauterine growth restriction.

A 19-year-old G1P0 woman notes vaginal spotting. Her last normal menstrual period occurred six weeks ago. She began having spotting early this morning and it has increased only slightly. She has no pain and denies other symptoms. Her medical history is noncontributory. Vital signs are: blood pressure 120/68; pulse 68; respirations 20; and temperature 98.6°F (37.0°C). On pelvic exam, her cervix is normal; uterus is small and nontender; and no masses are palpable. Initial labs show quantitative Beta-hCG 2000 mIU/ml and hematocrit 38%. A repeat Beta-hCG level 48 hours later is 2100 mIU/ml. A transvaginal ultrasound shows an empty uterus with a thin endometrial stripe and no adnexal masses. What is the next best step in the management of this patient? A. Dilation and curettage B. Treat with methotrexate C. Exploratory laparotomy D. Repeat Beta-hCG level in 48 hours E. . Repeat ultrasound in 24 hours

B. The patient clearly has an abnormal pregnancy, as demonstrated by the slowly increasing Beta-hCG levels. Since the Beta-hCG level is above 2000 mIU/ml, and she has a thin endometrial stripe, this excludes an intrauterine pregnancy and the diagnosis is an ectopic pregnancy. She is a good candidate for medical treatment with methotrexate. Criteria to consider for medical treatment include hemodynamic stability, non-ruptured ectopic pregnancy, and size of ectopic mass.

A 34-year-old G2P1 woman at 40 weeks gestation with a history of one prior vaginal delivery strongly desires an induction of labor, as she is unable to sleep secondary to severe back pain. Her cervical exam is closed, 20% effaced and -2 station. The cervix is firm and posterior. Which of the following is the most appropriate next step in the management of this patient? A. Wait until 42 weeks for induction B. Administer cytotec C. Insert a foley bulb in the cervix D. Perform artificial rupture of membranes E. Perform a Cesarean delivery

B. The patient is multiparous at term and waiting until she reaches 42 weeks may increase the risk of perinatal mortality. Since she is uncomfortable with back pain, it is reasonable to induce labor. Her cervix is unfavorable; therefore, cytotec administration is appropriate prior to pitocin induction. A foley bulb or artificial rupture of membranes cannot be achieved in a patient with a closed cervix. At this time, there are no indications to perform a Cesarean delivery in this patient.

A 32-year-old G1P0 woman comes to your office for her first prenatal care visit. She has recently read an article about the rising Cesarean section rate in the United States and asks you about the rate in your hospital. What do you explain as the major cause of higher Cesarean delivery rates? A. The rate of abnormal presentations has increased B. Less women are having vaginal births after Cesarean C. Obstetricians' reluctance to perform forceps delivery D. Increased rate of fetal macrosomia due to uncontrolled gestational diabetes E. Rate of twins has increased

B. The rate of vaginal birth after Cesarean (VBAC) has decreased in recent years due to studies that showed an increased risk of complications, especially uterine rupture. This is one factor that has led to the increased Cesarean section rate. In addition, although the rate of breech and other abnormal presentation is stable, there are significantly fewer obstetricians who are willing to perform vaginal breech deliveries (due in part to a lack of training in vaginal breech deliveries). Many obstetricians do not perform instrumental vaginal deliveries, such as forceps and vacuum extractions, further contributing to the rising rate. Gestational diabetes is a well-known pregnancy complication with clear clinical guidelines.

A 29-year-old G0 woman presents to your office for a routine visit. She has been trying to conceive for the last six months unsuccessfully. She requests fertility medications and hopes to get pregnant with twins. What counseling do you tell her regarding the risks of multifetal gestation? A. The morbidity with twin gestations is similar to triplet pregnancies B. The twin infant death rate is five times higher than that of singletons C. The rate of cerebral palsy is double in twin infants D. The incidence of abnormal fetal growth is similar to singleton pregnancies E. The incidence of prematurity is similar to singleton pregnancies

B. The twin infant death rate is five times higher than that of singletons. The epidemic of multiple gestations resulting from assisted reproductive techniques is of great significance to individual parturients and to society because of the major morbidities associated with twinning as well as with triplets and higher order multiples. The risk for development of cerebral palsy in twin infants is five to six times higher than that of singletons. One study, with dichorionic twins, monochorionic twins and singletons, showed that twins had a higher incidence of IUGR (intrauterine growth restriction) than singletons. Fifty-eight percent of twins deliver prematurely, with an average gestational age at delivery of 35 weeks. Twelve percent of twins deliver very prematurely.

A 24-year-old G3P0 woman at 26 weeks gestation was brought to the hospital by paramedics. Her husband found her shivering and barely responsive. Two days prior, the patient noted that she was feeling sick, with a slight cough. She was having back pain at the time, but thought it was probably normal for pregnancy. Her pregnancy has been uncomplicated except for the recent diagnosis of gestational diabetes. On exam, vital signs are: temperature 100.2°F (37.9°C); pulse 160; and blood pressure 68/32; respiratory rate 32. Oxygen saturation is 82% on room air. There is no apparent fundal tenderness, although the patient exhibits pain with percussion of the right back. Fetal heart tones are not audible. There is no evidence of vaginal bleeding. Extremities are cool to touch. White blood cell count 24,000; hemoglobin 9.5; hematocrit 27%. Urine microscopic analysis shows many white blood cells. What is the most likely etiology for this patient's disease? A. Abruptio placentae B. Pyelonephritis C. Diabetic ketoacidosis D. Chorioamnionitis E. Pneumonia

B. This is a patient in septic shock. The most common cause of sepsis in pregnancy is acute pyelonephritis. Given the absence of bleeding, the clinical picture is not suggestive of placental abruption. Diabetic ketoacidosis is unusual in gestational diabetic patients. Chorioamnionitis and pneumonia may both lead to sepsis, but are not suggested by the clinical picture.

A 17-year-old G2P0 female has severe right lower quadrant pain. Her last normal menstrual period was seven weeks ago. She notes that last night she began having suprapubic pain that radiated to her right lower quadrant. This morning, the pain awoke her from sleep. She has had no vaginal bleeding, no nausea or vomiting. The patient's history is notable for two first trimester elective abortions and a history of chlamydia treated twice. Vital signs are blood pressure 90/60; pulse 99; respirations 22; and temperature 98.6°F (37°C). On physical exam, the patient is noted to be curled on a stretcher in a fetal position and says she hurts too much to move. She has rebound and voluntary guarding on abdominal examination. She has severe cervical motion tenderness and rectal tenderness. Her Beta-hCG level is 2500 mIU/ml; hematocrit 24%; and urinalysis negative. Ultrasound shows no intrauterine pregnancy, a right adnexal mass that measures 6 x 2 cm, and a moderate amount of free fluid in the cul de sac. Which of the following is the most appropriate next step in the management of this patient? A. Admit for serial examinations B. Exploratory surgery C. Recheck Beta-hCG level in 48 hours D. Administer methotrexate E. Dilation and curettage

B. This patient has a ruptured ectopic pregnancy until proven otherwise. Her vital signs, examination and anemia are consistent with an intra-abdominal bleed. Exploratory laparoscopy/laparotomy is indicated at this point. Conservative management with observation, serial examinations or repeat Beta-hCG testing could be dangerous in a patient suspected of having a ruptured ectopic pregnancy. Medical management (methotrexate) is not used in a patient with an acute surgical abdomen. Dilation and curettage would not be the next step in management and might only be considered in this scenario after the patient's abdomen was explored.

A 29-year-old G3P0 woman presents for prenatal care at eight weeks gestation. Her two prior pregnancies ended in spontaneous losses at 19 and 18 weeks, respectively. Records corroborate the patient's history of an uncomplicated gestation until the evening of the losses, when she experienced a pink-tinged discharge that prompted her to call her obstetrician. In both cases, the obstetrician noted that her cervix had dilated completely with the amnionic sac bulging into the vagina to the level of the introitus. The patient was afebrile without other complaints and there was no uterine tenderness on exam. She spontaneously delivered the fetus and placenta in both cases. A sonohysterogram confirmed normal uterine anatomy several weeks later. What is the most appropriate next step in the management of this patient? A. Begin weekly fetal fibronectin testing B. Placement of a cervical cerclage at approximately 14 weeks gestation C. Immediate placement of a cervical cerclage D. Administer low dose aspirin and heparin E. Administration of prophylactic progesterone

B. This patient has an incompetent cervix and should have a cervical cerclage at 14 weeks. A positive fetal fibronectin does not indicate incompetent cervix and is used later in pregnancy as a negative predictor of preterm delivery. Pregnancy loss in the late second trimester is not usually related to genetic abnormality of the conceptus and most clinicians delay placement of a cerclage until after the first trimester, given the high background prevalence of first trimester pregnancy wastage. Although some clinicians strongly support the existence of an antiphospholipid antibody syndrome, the term most commonly refers to pregnancy loss or demise, rather than the clinical scenario of silent cervical dilation with delivery described. The patient would, therefore, not need aspirin or heparin. Although some clinicians use prophylactic progesterone to prevent recurrent abortion, as well as preterm labor, no controlled trials support the use of prophylactic progesterone in the treatment of cervical incompetence.

A 21-year-old G1 woman at 36 weeks gestation presents with sudden onset of abdominal pain and bleeding. She smokes a pack of cigarettes a day, but otherwise her pregnancy has been uncomplicated. She takes no medications other than prenatal vitamins. Her blood pressure is 150/90; pulse 90; and she is afebrile. Her uterus is tense and very tender. Pelvic ultrasound shows the placenta to be posterior and fundal, with a cephalic presentation of the fetus. Cervical examination reveals no lesions, blood coming through the os and is one centimeter dilated. Fetal heart tones have a baseline of 150, with a Category III fetal heart rate tracing. Tocometer reveals contractions every 30-45 seconds. Which of the following is the most appropriate next step in the management of this patient? A. Amniotomy B. Cesarean delivery C. Induction of labor D. Tocolysis E. A double set-up examination

B. This patient is undergoing a placental abruption, with a deteriorating fetal condition. An emergent Cesarean delivery is necessary. The mother risks excessive blood loss, DIC and possible hysterectomy. The fetus risks neurological injury from anoxia or death. Risk factors for abruption include smoking, cocaine use, abdominal trauma, chronic hypertension, multiparity and prolonged premature rupture of membranes. Since immediate delivery is needed, amniotomy, induction, or tocolysis are not appropriate. A double set-up examination (performed in the operating room with a Cesarean section team scrubbed and ready) is not indicated, since the ultrasound determined the location of the placenta to be fundal.

A 28-year-old G2P1 woman presents at 20 weeks gestation for a routine prenatal care visit. This pregnancy has been complicated by scant vaginal bleeding at seven weeks and an abnormal maternal serum alpha fetoprotein (MSAFP) with increased risk for Down syndrome, but normal amniocentesis: 46, XX. Her previous obstetric history is significant for a Cesarean delivery at 34 weeks due to placental abruption and fetal distress. Prenatal labs at six weeks showed blood type A negative, antibody screen positive: anti-D 1:64. Which of the following is the most likely cause of the Rh sensitization? A. ABO incompatibility B. Placental abruption C. Amniocentesis D. Abnormal maternal serum alpha fetoprotein (MSAFP) E. First trimester bleeding

B. This patient was sensitized during her first pregnancy that was complicated by abruption and required Cesarean delivery. Transplacental hemorrhage of fetal Rh-positive red blood cells into the circulation of the Rh-negative mother may occur following a number of obstetric procedures and complications, such as amniocentesis, chorionic villus sampling, spontaneous/threatened abortion, ectopic pregnancy, dilation and evacuation, placental abruption, antepartum hemorrhage, preeclampsia, Cesarean delivery, manual removal of the placenta and external version. ABO incompatibility is immune system reaction that occurs when blood from two different and incompatible blood types are mixed together.

A 31-year-old G3P0 woman at 27 weeks gestation is being managed expectantly for preeclampsia with severe features remote from term. Her blood pressure is 155/100 on methyldopa (Aldomet) 500 mg three times a day. Her recent 24-hour urine had 6.6 grams of protein. An ultrasound revealed a fetus with adequate growth, having an estimated fetal weight in the 10th percentile. Her labs are normal, except for a uric acid of 8.0 mg/dL; hematocrit 42% (increased from 37%); and platelet count 97,000. Which of these findings necessitates delivery at this time? A. Elevated uric acid B. Thrombocytopenia C. Proteinuria D. Poorly controlled blood pressures E. Hemoconcentration

B. Thrombocytopenia <100,000 is a contraindication to expectant management of severe preeclampsia remote from term (<32 weeks). Other contraindications include: inability to control blood pressure with maximum doses of two antihypertensive medications, non-reassuring fetal surveillance, liver function test elevated more than two times normal, eclampsia, persistent CNS (central nervous system) symptoms and oliguria. Delivery should not be based on the degree of proteinuria. Although elevated, uric acid and hemoconcentration are markers of preeclampsia, they are not part of the diagnostic or management criteria.

A 19-year-old G2P1 woman presents with vaginal spotting and uterine cramping. Her last normal menstrual period was six weeks ago and she began spotting three days ago. She has no history of sexually transmitted infections. Her vital signs are blood pressure 120/70; pulse 78; respirations 20; and temperature 98.6°F (37.0°C). On pelvic examination, she has no cervical motion tenderness, her uterus is normal size and non-tender; no adnexal masses are palpable. Quantitative Beta-hCG 48 hours ago was 1500 mIU/ml. Currently, Beta-hCG is 3100 mIU/ml. Progesterone is 26 ng/ml; hematocrit is 38%; and urinalysis is normal. What is the most likely finding on transvaginal ultrasound? A. Debris in uterus B. Viable intrauterine pregnancy C. Adnexal mass, empty uterus D. No adnexal mass, empty uterus E. Non-viable intrauterine pregnancy

B. Transvaginal ultrasound will most likely show an intrauterine pregnancy. The Beta-hCG level is above the discriminatory zone for ultrasound (2000 mIU/ml), and the level has doubled in 48 hours. Additionally, the progesterone level is within expected range for a normal pregnancy (>25 ng/ml suggests healthy pregnancy) and up to 30% of all normal pregnancies experience first trimester spotting/bleeding. The findings of debris in the uterus, an empty uterus, with or without an adnexal mass, or free fluid (suggesting hemoperitoneum) would not be anticipated.

A 35-year-old G1P0 woman with a known twin gestation undergoes an ultrasound evaluation at 18 weeks. She would like to know if her twins are identical or fraternal. Which ultrasound marker is suggestive of dizygotic (fraternal) twins? A. Increased amniotic fluid volume in one of the twins B. Two separate placentas (anterior and posterior) C. Dividing membranes less than 1 mm D. Concordant growth of the twins E. Twin-twin transfusion syndrome

B. Ultrasound markers suggestive of dizygotic (non-identical) twins include a dividing membrane thickness greater than 2 mm, twin peak (lambda) sign, different fetal genders and two separate placentas (anterior and posterior). The two different placental types in twin gestation are monochorionic and dichorionic. Dizygotic conceptions always have dichorionic placentas. Monozygotic conceptions may have either monochorionic or dichorionic placentation, depending upon the time of division of the zygote. Diamniotic dichorionic placentation occurs with division prior to the morula state (within three days post fertilization). Diamniotic monochorionic placentation occurs with division between days four and eight post-fertilization. Monoamniotic, monochorionic placentation occurs with division between days eight and 12 post fertilization. Division at or after day 13 results in conjoined twins. The ultrasound markers for determination of chorionicity described above have been used to assess risk for complications of pregnancy, most notably the twin-twin transfusion syndrome.

A 29-year-old G4P2 woman was diagnosed with twin-twin transfusion syndrome when an ultrasound was performed at 24 weeks gestational age. Which of the following is a complication of twin-twin transfusion syndrome? A. Fetal macrosomia in the donor twin B. Neurologic sequelae in the surviving twin C. Tricuspid regurgitation in the donor twin D. Heart failure in the donor twin only E. High perinatal mortality for donor twin only

B. Untreated severe twin-twin transfusion syndrome has a poor prognosis, with perinatal mortality rates of 70-100%. Death in utero of either twin is common. Surviving infants have increased rates of neurological morbidity, including an increased risk of cerebral palsy. Excessive volume can lead to cardiomegaly, tricuspid regurgitation, ventricular hypertrophy and hydrops fetalis for the recipient twin. Although the recipient twin is plethoric, it is not macrosomic. The donor twin becomes anemic and hypovolemic, and growth is retarded. The recipient twin becomes plethoric and hypervolumic. Either twin can develop hydrops fetalis. The donor twin can become hydropic because of anemia and high-output heart failure.

A 22-year-old G1P0 woman currently at 38 weeks gestation complains of decreased fetal movement. Ultrasound reveals a 38-week gestation with no cardiac activity. The parents are upset and want to know the next steps. Which of the following would be most helpful to aide the bereavement process? A. Minimize analgesia used during labor B. Plan for postpartum care on the maternity floor C. Allow the parents to decide when to deliver D. Immediately remove the infant from the room after delivery E. Avoid requesting an autopsy and stillbirth evaluation

C. Allowing the parents when to decide to deliver can help the bereavement process. Keeping the patient adequately anesthetized during the labor and delivery as well as letting the parents hold the baby for as long as they desire also helps them grieve. Whether to have care on the maternity floor needs to be the parents' decision as well. Offering an autopsy to determine the cause of death as well as having someone taking pictures and keeping mementos for the parents is helpful.

A 24-year-old G2P1 woman at 18 weeks gestation with a history of asthma presents to the office with worsening symptoms, needing to use her inhaler more frequently. The symptoms began with the pregnancy and have gradually increased. She is using her albuterol inhaler as needed, recently three times a day. She denies any illness or fever. She has had asthma since she was a child. On exam, the patient appears comfortable. Her temperature is 100.2°F (37.9°C) and respiratory rate is 18. Auscultation of the lungs shows good air movement with mild scattered end expiratory wheezes. There are no rales or bronchial breath sounds. Which of the following is the most appropriate next step in the management of this patient? A. Oral theophylline B. Subcutaneous terbutaline C. Inhaled corticosteroids D. Oral zafirlukast (leukotriene inhibitor) E. Antibiotic treatment

C. Asthma generally worsens in 40% of pregnant patients. One of the indications for moving to the next line of treatment includes the need to use beta agonists more than twice a week. The appropriate choice for her treatment would be inhaled corticosteroids or cromolyn sodium. Theophylline would be used in more refractory patients. Subcutaneous terbutaline and systemic corticosteroids would be used in acute cases. Zafirlukast, a leukotriene receptor antagonist, is not effective for acute disease. The safety of zafirlukast in pregnancy is not well established because there is little experience with their use in pregnancy. Antibiotic treatment is only used when a pulmonary infection is diagnosed.

A 22-year-old G1 woman is undergoing treatment with magnesium sulfate for preeclampsia with severe features. She was delivered 10 hours ago via Cesarean section for a non-reassuring fetal heart rate tracing. She has oliguria and appears lethargic. On exam, no deep tendon reflexes can be appreciated. Her magnesium level is 11 mEq/L. Which of the following conditions is most likely to occur in this patient? A. Seizures B. Paralysis C. Respiratory depression D. Pulmonary edema E. Cardiac arrest

C. At a magnesium level of 11 mEq/L, respiratory depression is most likely to occur. The therapeutic magnesium level is between 4-7 mEq/L. Seizures are prevented by the use of magnesium. Loss of deep tendon reflexes occurs at a level of 7-10 mEq/L. Cardiac arrest may occur at a level of 15 mEq/L.

A 25-year-old G1 woman at 41 weeks gestation presents to labor and delivery with painful contractions every four minutes. Her cervix is 5 cm dilated, 90% effaced. On cervical exam, you are able to feel a fetal body part but it is not the head. Which of the following is the most likely body part you were palplating? A. Foot B. Hand C. Buttocks D. Back E. Shoulder

C. Breech presentation occurs in approximately 3-4% of women in labor overall, and occurs more frequently in preterm deliveries. Frank breech is the most common type, occurring in 48-73% of cases and the buttocks are the presenting part. Complete breech is found in approximately 5-12% of cases and incomplete breech (footling breech) occurs in approximately 12-38% of cases.

A 17-year-old G1 woman at 24 weeks gestation presents with vaginal bleeding. She denies any pain, cramping or dysuria. She reports last having intercourse three weeks ago. Prenatal care and labs have been unremarkable. Her vital signs are normal and she is afebrile. Pelvic ultrasound reveals a fundal placenta and viable fetus. Abdominal examination is unremarkable. Vaginal examination reveals a uniformly friable cervix with a small amount of blood in the vault. Digital examination reveals a firm, closed cervix. What is the most likely diagnosis that explains the bleeding? A. Trauma B. Cervical cancer C. Cervicitis D. Bloody show E. Threatened abortion

C. Cervicitis caused by chlamydia, gonorrhea, trichomonas or other infections can present with vaginal bleeding. The cervix is much more vascular during pregnancy and inflammation can lead to bleeding. Evaluation for other causes of bleeding must be completed and then treatment for the infection should be initiated. The patient does not give any history of trauma and cancer is unlikely because of her age. She is not in labor, and a bloody show associated with cervical dilatation is not consistent with the history provided. Threatened abortion occurs during the first trimester.

A 25-year-old G1 woman at term presents in active labor. Her cervix rapidly changes from 7 centimeters to complete dilation in 1 hour. She has been pushing for two hours. The fetal station has changed from -1 to +1. Fetal heart tracing is Category I. The patient is feeling strong contractions every three minutes. Which of the following is the most appropriate next step in the management of this patient? A. Cesarean delivery B. Forceps delivery C. Continued monitoring of labor D. Augmentation with oxytocin E. Ultrasound for estimated fetal weight

C. Continued monitoring of labor is appropriate if clinical evaluation indicates that the fetus is not macrosomic or there is no obvious fetopelvic disproportion. If either were the case, then a Cesarean delivery would be indicated. At this time, there is no fetal or maternal indication to perform a forceps delivery because the station is +1. Augmentation would be indicated if the contractions were inadequate in intensity or frequency. An ultrasound at this stage of labor is inaccurate and one relies on clinical estimates of weight.

A 32-year-old G3P2 woman presents at 40 1/7 weeks gestation because of regular uterine contractions every five minutes for the last two hours. Her prenatal course was unremarkable. She states the baby is moving, but she has had a bright red, bloody discharge for the last 30 minutes. She does not think she has ruptured her membranes. Her blood pressure is 120/70; pulse 80; and she is afebrile. Her abdomen is soft and she has regular contractions of moderate intensity. Fetal heart tones have a baseline of 130 with a Category I fetal heart rate tracing. Pelvic ultrasound reveals a fundal placenta and cephalic presentation of the fetus. Cervical examination reveals a friable cervix that bleeds easily and is 5 centimeters dilated and completely effaced. Membranes are confirmed to be intact. Which of the following is the most likely source of bleeding? A. Placental abruption B. Placenta previa C. Bloody show D. Cervical cancer E. Cervicitis

C. During pregnancy the cervix is extremely vascular, and with dilation a small amount of bleeding may occur. This bloody show is not of clinical significance and often occurs with normal labor. Serious causes of bleeding, such as placental abruption and placenta previa, need to be ruled out in order to make the proper delivery plans. Cervical cancer and cervicitis are very unlikely causes for the bleeding in this situation.

A 25-year-old G2P1 woman at eight weeks gestation is diagnosed with a spontaneous abortion. Her husband is 40 years old. The patient's past medical history is noncontributory. She gets some exercise regularly and smokes two packs of cigarettes a day. Three years ago, she had a full-term delivery that was complicated by mild preeclampsia. Which of the following factors is most likely the cause of this spontaneous abortion? A. Infection B. Advanced paternal age C. Environmental factors D. Uterine anomaly (i.e. unicornuate uterus) E. History of preeclampsia

C. Environmental factors, such as smoking, alcohol and radiation are causes of spontaneous abortion. Although the risk increases with infections, such as listeria, mycoplasma, ureaplasma, toxoplasmosis and syphilis, advancing maternal or paternal age, advancing parity and some mullerian anomalies, the clinical scenario does not support these as possible causes. An isolated history of preeclampsia confers no increase in risk of spontaneous abortion.

A 23-year-old G2P0 woman at 33 weeks gestation presents to labor and delivery with acute nausea, vomiting and epigastric pain. Her blood pressure is 145/90; she has 1+ protein on a urinalysis. Her labs are shown below: Hematocrit: 42% White blood cell count: 11,000/mcL Bilirubin: 1.4 mg/dL Platelets: 42,000/mcL Lipase: 11 u/L Aspartate aminotransferase (AST): 391 u/L Creatinine: 0.8 mg/dL Alanine aminotransferase (ALT): 444 u/L Uric acid: 7.7 mg/dL Glucose: 100 mg/dL Fibrinogen: 405 mg/dL Which of the following is the most likely diagnosis in this patient? A. Mild preeclampsia B. Hepatitis C. HELLP syndrome D. Cholecystitis E. Acute fatty liver

C. HELLP syndrome is a disease process in the spectrum of severe preeclampsia. The acronym stands for "hemolysis, elevated liver enzymes, low platelets" and can lead to swelling of the liver capsule and possibly liver rupture. It may or may not be accompanied by right upper quadrant pain. It is possible to only have thrombocytopenia and elevated transaminases without clear hemolysis (elevated bilirubin and anemia), especially if a diagnosis is made early. This patient does not have seizures and, therefore, does not have eclampsia. The clinical scenario is not consistent with hepatitis or cholecystitis. Acute fatty liver almost always manifests late in pregnancy. Symptoms develop over several days to weeks and include malaise, anorexia, nausea and vomiting, epigastric pain, and progressive jaundice. In many women, persistent vomiting in late pregnancy is the major symptom. About half of all women have hypertension, proteinuria, and edema signs suggestive of preeclampsia. There is usually severe liver dysfunction with hypofibrinogenemia, hypoalbuminemia, hypocholesterolemia, and prolonged clotting times. As acute fatty liver worsens there is marked hypoglycemia.

A 32-year-old G3P0 woman presents to the clinic for preconception counseling. Her prior three pregnancies resulted in first trimester losses. Which of the following tests should be ordered for this patient? A. Adrenal stimulation test B. Clomiphene citrate-FSH challenge test C. Lupus anticoagulant test D. Pelvic MRI E. CT scan of the pelvis

C. It is important to rule out systemic disease in a patient with recurrent abortion (three successive first trimester losses). Testing for lupus anticoagulant, diabetes mellitus and thyroid disease are commonly performed. Maternal and paternal karyotypes should also be obtained. Infectious causes should also be considered. Uterine imaging to exclude a septum or other anomaly is routinely done using hysteroscopy or hysterography and not CT or MRI scanning. There is no role for clomiphene citrate-FSH challenge in the evaluation of this patient.

A 38-year-old G4P2 woman was diagnosed with triplets when an ultrasound was performed at 12 weeks gestational age. Which of the following is the most concerning complication for this multiple gestation? A. Preeclampsia B. Intrauterine growth restriction C. Preterm birth D. Gestational diabetes E. Abnormal placentation

C. Preterm delivery increases the risk of morbidity and mortality, increasing with higher orders of multiples. Preterm birth occurs in over 50% of twin pregnancies, 90% of triplet pregnancies, and almost all quadruplet pregnancies. While all the choices may occur with a multiple gestation, prematurity has the most significant consequences as it is associated with an increased risk of respiratory distress syndrome (RDS), intracranial hemorrhage, cerebral palsy, blindness, and low birth weight. Intrauterine growth restriction, intrauterine death of one or more fetuses, miscarriage and congenital anomalies are all more common with multiple gestations, as are the complications of preeclampsia, diabetes and placental abnormalities.

A 19-year-old G1P0 woman with a desired pregnancy notes vaginal spotting early this morning and it has slightly increased. Her last normal menstrual period occurred six weeks ago. She has no pain or other symptoms. Her medical history is noncontributory. Vital signs are: blood pressure 120/68; pulse 68; respirations 20; and temperature 98.6°F (37.0°C). On pelvic exam, her cervix is normal; her uterus is small and nontender; there are no masses palpable. Labs show: quantitative Beta-hCG 750 mIU/ml; progesterone 3.8 ng/ml; hematocrit 38%. Which of the following is the most appropriate next step in the management of this patient? A. Order a transvaginal ultrasound B. Repeat Beta-hCG level in 24 hours C. Repeat Beta-hCG level in 48 hours D. Dilation and curettage E. Bed rest

C. Repeating the Beta-hCG level will show whether the pregnancy is viable or failing. The appropriate time interval for repeating the initial level is 48 hours, since during the first 42 days of gestation levels increase by approximately 50% every 48 hours in most viable pregnancies. Ordering an ultrasound would not be helpful, since the patient's Beta-hCG level is lower than the discriminatory zone (the level at which an intrauterine pregnancy should be seen on ultrasound, usually 2000 mIU/ml). There is no need to repeat the progesterone level. Dilation and curettage or treatment with methotrexate are both inappropriate without a diagnosis, since both could interrupt a viable pregnancy. Bedrest is not indicated in this patient.

A 25-year-old G0 woman presents to her doctor for preconception counseling. She is healthy without significant medical problems. She takes no medications. She smokes one pack of cigarettes per day since age 16 and drinks occasionally. She weighs 140 pounds and her vital signs and examination are normal. The patient is at increased risk of which of the following during her pregnancy? A. Fetal chromosomal abnormality B. Breech presentation C. Placental abruption D. Cerebral palsy E. Neural tubal defect

C. Smoking increases the risk of several serious complications of pregnancy, including placental abruption, placenta previa, fetal growth restriction, preeclampsia and infection. Women who smoke should be counseled vigorously to quit smoking prior to conception and to resist restarting after the baby is born.

A 27-year-old G2P0 woman is diagnosed with an early first trimester spontaneous abortion. She has a history of type 1 diabetes mellitus, mild chronic hypertension and one prior termination of pregnancy. Which of the following is the most likely cause of this spontaneous abortion? A. Prior termination of pregnancy B. Chronic hypertension C. Diabetes mellitus D. Intrauterine adhesions E. Infection

C. Systemic diseases such as diabetes mellitus, chronic renal disease and lupus are associated with early pregnancy loss. In women with insulin-dependent diabetes, the rates of spontaneous abortion and major congenital malformations are both increased. The risk appears related to the degree of metabolic control in the first trimester. There are many other causes of spontaneous abortion, including genetic factors, endocrine abnormalities, reproductive tract abnormalities, immunologic factors and environmental factors. The patient's history of mild chronic hypertension and one prior termination of pregnancy do not increase her risk of a first trimester loss. Additionally, an uncomplicated termination of pregnancy, intrauterine adhesions and infection are not likely causes in this scenario.

A 22-year-old G4P1 woman at 26 weeks gestation presents complaining of a postcoital musty odor and increased milky, gray-white discharge for the last week. This was an unplanned pregnancy. She had her first pregnancy at age 15. She reports that she has no new sex partners, but the father of the baby may not be monogamous. On examination, there is a profuse discharge in the vaginal vault, which covers the cervix. Pertinent labs: wet mount pH >4.5 and whiff test positive. Microscopic exam reveals clue cells, but no trichomonads or hyphae. Which of the following is the most appropriate next step in the management of this patient? A. Delay treatment until postpartum B. Treat her now and again during labor C. Treat her now D. Treat her and her partner E. No treatment necessary

C. The patient has bacterial vaginosis. All symptomatic pregnant women should be tested and treatment should be not be delayed because treatment has reduced the incidence of preterm delivery. The optimal regimen for women during pregnancy is not known, but the oral metronidazole regimens are probably equally effective. Once treated antepartum, there is no need to treat during labor unless she is reinfected.

A 30-year-old G3P2 woman, whose last normal menstrual period was eight weeks ago, began spotting three days ago and developed cramping this morning. She has a history of a chlamydia infection with a previous pregnancy. She smokes one pack of cigarettes per day and denies alcohol or drug use. On physical exam: blood pressure 120/70; pulse 82; respirations 20; and temperature 98.6°F (37.0°C). Abdominal examination is normal. Pelvic examination reveals old blood in the vaginal vault, closed cervix without lesions, slightly enlarged uterus and no adnexal tenderness. Pertinent labs: quantitative Beta-hCG is 1000 mIU/ml; urinalysis normal; hematocrit = 32%. Transvaginal ultrasound shows no intrauterine pregnancy, no adnexal masses, and no free fluid in pelvis. Which of the following is the most appropriate next step in the management of this patient? A. Treat with methotrexate B. Exploratory surgery C. Repeat Beta-hCG in 48 hours D. Repeat Beta-hCG in one week E. Admit the patient to the hospital for observation

C. The patient has risk factors for ectopic pregnancy, but needs an accurate diagnosis before a treatment plan is entertained. Repeating the Beta-hCG is the next step in this patient's management. Inappropriately rising Beta-hCG levels (less than 50% increase in 48 hours) or levels that either do not fall following diagnostic dilation and curettage would be consistent with the diagnosis of ectopic pregnancy. Alternatively, a fetal pole must be visualized outside the uterus on ultrasound. The patient would need a Beta-hCG level over the discriminatory zone (the level where an intrauterine pregnancy can be seen on ultrasound) with an empty uterus. The level commonly used is 2000 mIU/ml. Treatment with methotrexate may be appropriate, but only after a definitive diagnosis is made. The patient does not yet have this level and is stable. She is, therefore, not a candidate for exploratory surgery. If she had unstable vital signs or an acute abdomen, a diagnostic laparoscopy/laparotomy would be indicated. Repeating the ultrasound in one week is not recommended because a delay in diagnosis could result in a ruptured ectopic pregnancy and increased risk to the patient. The patient is hemodynamically stable; therefore, she does not need to be admitted to the hospital.

A 20-year-old G1 woman at 40 weeks gestation presents to labor and delivery complaining of painful contractions every 3-4 minutes for the last eight hours. Cervical examination on admission was 2 centimeters dilated, 90% effaced and 0 station. Three hours later, her exam is unchanged. The patient is still having contractions every 3-4 minutes. She is discouraged about her lack of progress. Which of the following is the most appropriate next step in the management of this patient? A. Laminaria placement B. Artificial rupture of membranes C. Counseling about latent phase of labor and rest D. Manual cervical dilation E. Cesarean section for arrest of labor

C. The patient is in the latent phase of labor and has not yet reached the active phase (more than 4 cm). A prolonged latent phase is defined as >20 hours for nulliparas and >14 hours for multiparas, and may be treated with rest or augmentation of labor. Artificial rupture of membranes is not recommended in the latent phase as it places the patient at increased risk of infection. Cervical dilation or laminaria placement are not indicated.

A 16-year-old G1P0 African-American female presents at eight weeks gestation for prenatal care. She reports occasional spotting but denies pain or fever. The laboratory reports hemoglobin of 8 g/dL and a peripheral smear reveals hypochromia and microcytosis. Which of the following is the most likely diagnosis for this patient? A. Sickle cell anemia B. Folate deficiency C. Iron deficiency D. β-thalassemia E. Acute blood loss

C. The two most common causes of anemia during pregnancy and the puerperium are iron deficiency and acute blood loss. Classical morphological evidence of iron-deficiency anemia includes erythrocyte hypochromia and microcytosis. Serum ferritin levels are lower than normal and there is no stainable bone marrow iron on examination of a bone marrow aspirate. The spotting she reports would not lead to anemia due to blood loss

A 33-year-old G3P1 woman presents with left lower quadrant pain of two days duration and seven weeks of amenorrhea. She describes her pain as mild and intermittent. Past medical history is significant for smoking during her teens, ectopic pregnancy with salpingostomy four years ago, multiple Chlamydia infections in her teens, and an uncomplicated Cesarean delivery for breech presentation. Which of the following risk factors is most likely associated with recurrent ectopic pregnancy in this patient? A. Age B. History of chlamydia infections C. History of ectopic pregnancy D. Prior Cesarean delivery E. Interval between pregnancies

C. There is a tenfold increase risk for ectopic pregnancy in women with a prior history of ectopic pregnancy. Age between 35 and 44 years old is associated with a threefold increase in ectopic pregnancy. Prior abdominal surgery and history of sexually transmitted infections as well as sterilization failures, endometriosis and congenital uterine malformations are all associated with an increased risk of ectopic pregnancy. The interval between pregnancies and past smoking history is not associated with an increased risk of ectopic pregnancy.

A 24-year-old G1P0 at 32 weeks gestation presents with vaginal bleeding most likely caused by placental abruption. She receives a standard dose of 300 micrograms of RhoGAM. What amount of fetal blood is neutralized by this dose? A. 10 cc B. 20 cc C. 30 cc D. 40 cc E. 50 cc

C. Thirty (30) cc of fetal blood is neutralized by the 300 micrograms dose of RhoGAM. This is equivalent to 15 cc of fetal red blood cells. At 28-weeks gestation, 300 micrograms of Rh-immune globulin is routinely administered after testing for sensitization with an indirect Coombs' test. Administration is given following amniocentesis at any gestational age.

A 27-year-old G1 woman at 36 weeks gestation is undergoing an induction of labor for preeclampsia with severe features. She complains of a headache, right upper quadrant pain and seeing spots. Admission vital signs are: blood pressure 180/120, respiratory rate 20, pulse 92. In addition to 10 hours of oxytocin, she is receiving intravenous magnesium sulfate 2 g /hour. During the past two hours her urine output has decreased to 15 mL per hour (down from 40 mL/h) and her respiratory rate is now 10. What is the next best step in the management of this patient? A. Continue magnesium sulfate B. Decrease magnesium sulfate to 1 gram/hour C. Administer calcium gluconate D. Increase oxytocin drip E. Cesarean delivery now

C. This patient has respiratory depression likely secondary to magnesium toxicity. In addition to discontinuing the magnesium sulfate, she needs a dose of calcium gluconate to restore her respiratory function. The classic signs of magnesium toxicity include muscle weakness and loss of deep tendon reflexes, nausea, and respiratory depression. If magnesium is given in high doses, cardiac arrest is possible. Altering the oxytocin drip and a Cesarean delivery are not indicated at this time.

A 30-year-old G2P1 woman at 38 weeks gestation presents to labor and delivery with contractions every 2-3 minutes. Her membranes are intact. Her cervical examination is 5 centimeters dilated, 100% effaced, and -1 station. The fetal heart rate tracing is Category I. Two hours later, she progresses to 7 cm and 0 station and receives an epidural for pain. Four hours after that, her exam is unchanged (7/100/0). Fetal heart rate tracing remains Category I. Which of the following is the most appropriate next step in the management of this patient? A. Allow her to ambulate and return when she is ready to push B. Perform a contraction stress test C. Perform an amniotomy D. Perform a Cesarean delivery E. Place an internal fetal scalp electrode

C. This patient has secondary arrest of dilation, as she has not had any further cervical change in the active phase for over four hours. Amniotomy is often recommended in this situation. After it is performed, if the patient is still not in an adequate contraction pattern, augmentation with oxytocin can be attempted after careful evaluation. Although the patient requires close monitoring, it is too early to proceed with a Cesarean delivery. An internal scalp electrode is not necessary, since the fetal heart monitoring is reassuring.

An 18-year-old G1 woman presents for prenatal care at 16 weeks gestation without complaints. The patient denies any history of sexually transmitted disease, although admits to a history of multiple sex partners, with irregular use of condoms. She is allergic to penicillin, which causes anaphylaxis. Physical exam is unremarkable. Pertinent labs: rapid plasma reagin test (RPR) positive (titer = 32); fluorescent treponemal antibody absorption test (FTA-ABS) is positive. Which of the following is the best treatment for this patient? A. Oral erythromycin B. Oral doxycycline C. Desensitization and penicillin D. Intravenous erythromycin E. Intravenous cefazolin

C. This patient has syphilis, and the fluorescent treponemal antibody absorption test (FTA-ABS) confirms the diagnosis. The transmission rates for primary and secondary disease are approximately 50-80%. There are no proven alternatives to penicillin therapy during pregnancy and penicillin G is the therapy of choice to treat syphilis in pregnancy. Women with a history of penicillin allergy can be skin tested to confirm the risk of immunoglobulin E (IgE)-mediated anaphylaxis. If skin tests are reactive, penicillin desensitization is recommended and is followed by intramuscular benzathine penicillin G treatment. Erythromycin has an 11% failure rate. Doxycycline is contraindicated in pregnancy. Cefazolin is commonly used to treat urinary tract infections and is not effective in the treatment of syphillis.

A 22-year-old G1P0 woman presents to the emergency department at eight weeks gestation experiencing heavy vaginal bleeding. On physical exam: blood pressure 94/60; pulse 108; respirations 20; and temperature 98.6°F (37.0°C). Pelvic examination demonstrates brisk bleeding through a dilated cervical os. The patient's hemoglobin is 7 g/dL (hematocrit 21%). Which of the following is the most appropriate next step in the management of this patient? A. Administration of intravaginal misoprostol B. Administration of oral misoprostol C. Dilation and suction curettage D. Endometrial ablation E. Expectant care to permit spontaneous abortion

C. This patient is actively bleeding and is anemic. She, therefore, requires immediate surgical treatment consisting of dilation and suction curettage. Although clinicians increasingly utilize both expectant management and various drug regimens to treat spontaneous abortion, a prerequisite for either is that the patient is hemodynamically stable and reliable for follow-up care. She is not hemodynamically stable. Endometrial ablation will not work in this case, as the products of conception need to be evacuated to control the bleeding.

A 23-year-old G2P1 woman at 36 weeks gestation presents with her third episode of heavy vaginal bleeding. She has normal prenatal labs and a known placenta previa. She denies uterine contractions or abdominal pain and reports good fetal movement. Her vital signs are: blood pressure 100/60; pulse 110; and she is afebrile. Her abdomen and uterus are non-tender. Fundal height measures 35 centimeters and fetal heart tones reveal a baseline of 140 and are reassuring. Pelvic ultrasound confirms a placenta previa and the fetus is in the cephalic presentation. Hematocrit is 29%. Which of the following is the most appropriate next step in the management of this patient? A. Tocolysis B. Induction of labor C. Cesarean delivery D. Amniocentesis E. Administer steroids

C. This patient is near term with a third episode of active bleeding from a placenta previa. The appropriate next step would be to move towards delivery via Cesarean section. The patient is not experiencing contractions, so tocolysis is not necessary and would not be used with heavy vaginal bleeding. Catastrophic bleeding could occur due to disruption of blood vessels as the cervix dilates if a vaginal delivery is pursued, and induction of labor would therefore be contraindicated. An amniocentesis is not indicated in this situation. Although the patient is not yet at term, delivery is appropriate due to the third episode of heaving bleeding at near term. Administering steroids is not appropriate at this gestational age.

A 39-year-old G1P0 woman at 37 weeks gestation is being induced secondary to polyhydramnios. She has received cervical ripening with prostaglandin E2, and is now on oxytocin. Her cervix on last check was 3cm, 75% effaced, -4 station. Her water just broke, and the nurse reports that the patient is now contracting every minute and notes that she is having a large amount of bleeding. On exam, her blood pressure is 100/80; pulse 100; temperature of 100.0°F (37.8°C); and she has a Category II tracing. Her cervical exam is 10cm, 100% effaced, +2 station. Which of the following is the most likely explanation for her bleeding and labor progression? A. Oxytocin B. Rupture of membranes C. Abruptio placentae D. Chorioamnionitis E. Normal labor progression

C. This patient likely is experiencing a placenta abruption. Her biggest risk factor is polyhydramnios with rapid decompression of the intrauterine cavity. While oxytocin is used to augment labor, in a nulliparous patient, the difference between her two exams is extreme. Typically, normal labor progresses about 1 cm per hour in the active phase of labor (multiparous woman about 1-2 cm/hour). While there may be some vaginal bleeding (bloody show, or light bleeding with cervical dilation) it is not normal to have a large amount of bleeding. This patient has no signs or symptoms of chorioamnionitis.

A 29-year-old G4P2 woman with no previous prenatal care presents at 24 weeks gestation with signs and symptoms of preterm labor. Her cervix is 3 cm dilated and 80% effaced. Fundal height is 30 cm and an ultrasound examination reveals a twin gestation. Estimated fetal weights on the twins are 850 gm and 430 gm. The maximum vertical amniotic fluid pocket around the smaller twin is 1 cm; the maximum vertical amniotic fluid pocket around the larger twin is 8 cm. Which of the following is the most likely associated with these ultrasound findings? A. Dichorionic diamniotic twins B. Monochorionic monoamniotic twins C. Monochorionic diamniotic twins D. Superfecundation E. Rh-isoimmunization

C. Twin-twin transfusion syndrome is the result of an intrauterine blood transfusion from one twin to the other. It most commonly occurs in monochorionic, diamniotic twins. The donor twin is often smaller and anemic at birth. The recipient twin is usually larger and plethoric at birth. Clues to the presence of the twin-twin transfusion syndrome include the large weight discordance (although this is not necessary for diagnosis), polyhydramnios around the larger (recipient) twin, and oligohydramnios around the smaller (pump) twin. The two different placental types in twin gestation are monochorionic and dichorionic. Monozygotic conceptions may have either monochorionic or dichorionic placentation, depending upon the time of division of the zygote. Dizygotic conceptions always have dichorionic placentas. Diamniotic dichorionic placentation occurs with division prior to the morula state (within three days post fertilization). Diamniotic monochorionic placentation occurs with division between days four and eight post fertilization. Monoamniotic, monochorionic placentation occurs with division between days eight and 12 post fertilization. Superfecundation is the fertilization of two different ova at two separate acts of intercourse in the same cycle. Isoimmunization is associated with polyhydramnios and fetal hydrops and does not cause twin-twin transfusion.

A 24-year-old G2P1 woman has a fetus that is affected by Rh disease. At 30 weeks gestation, the delta OD450 (optical density deviation at 450 nm) results plot on the Liley curve in Zone 3, indicating severe hemolytic disease. Which of the following is the most appropriate next step in the management of this patient? A. Immediate Cesarean delivery B. Induction of labor C. Intrauterine intravascular fetal transfusion D. Umbilical blood sampling E. Maternal plasmapheresis

C. Values in Zone 3 of the Liley curve indicate the presence of severe hemolytic disease, with hydrops and fetal death likely within 7-10 days, thus demanding immediate delivery or fetal transfusion. At 30 weeks gestation, the fetus would benefit from more time in utero. An attempt should be made to correct the underlying anemia. Intravascular transfusion into the umbilical vein is the preferred method. Intraperitoneal transfusion is used when intravascular transfusion is technically impossible. If fetal hydrops is present, the reversal of the fetal anemia occurs much more slowly via intraperitoneal transfusion. Percutaneous umbilical blood sampling should not be used as a first-line method to evaluate fetal status. Maternal plasmapheresis is used in severe disease when intrauterine transfusions are not possible.

A 36-year-old G5P4 woman with no prenatal care presented in active labor with a blood pressure of 170/105 and 3+ proteinuria. Fundal height is 28 cm. Fetal heart tones were found to be in the 170s with decreased variability and a sinusoidal pattern. Resting uterine tone was noted to be increased and she was having frequent contractions (every 1-2 minutes). The patient complained of bright red vaginal bleeding for the past hour. Based on this history, what is the most likely etiology of her vaginal bleeding? A. Uterine rupture B. Placenta previa C. Bloody show D. Abruptio placentae E. Cervical trauma

D. Although all the options above can result in third trimester vaginal bleeding, the most likely cause in this patient is placental abruption. This diagnosis goes along with the tachysystole on tocometer and evidence of fetal anemia (tachycardia and sinusoidal heart rate pattern) on the heart rate tracing. Hypertension and preeclampsia are risk factors for abruption. She has no history of cervical trauma.

A 36-year-old G2P1 woman presents for her first prenatal visit at 11 weeks gestation. She has a two-year history of chronic hypertension treated with lisinopril and labetalol. In addition, she has hypothyroidism treated with levothyroxine, and recurrent herpes, for which she is on chronic acyclovir suppressive therapy. She takes amitriptyline for migraine headaches. Which of her medications is contraindicated in pregnancy? A. Levothyroxine B. Labetalol C. Acyclovir D. Lisinopril E. Amitriptyline

D. Amitriptyline, levothyroxine, labetalol and acyclovir are medications that are frequently used in pregnancy and are felt to have acceptable safety profiles. The use of angiotensin converting enzyme inhibitors, such as Lisinopril, beyond the first trimester of pregnancy has been associated with oligohydramnios, fetal growth retardation and neonatal renal failure, hypotension, pulmonary hypoplasia, joint contractures and death. Amitriptyline is used in pregnancy to treat migraine headaches.

A 33-year-old G2P1 woman at eight weeks presents to the clinic. This is an unplanned pregnancy. She had planned a tubal ligation six years ago when she was diagnosed with pulmonary hypertension, but was unable to have the procedure. She states her pulmonary hypertension has been stable, but she gets short of breath when climbing stairs. She sleeps on one pillow at night. What is the concern for her during this pregnancy? A. There are no additional concerns compared to a normal pregnancy B. She will need a Cesarean section at delivery C. Her baby is at increased risk for pulmonary hypoplasia D. The mother's mortality rate is above 25% E. Epidural analgesia is contraindicated

D. Among women with cardiac disease, patients with pulmonary hypertension are among the highest risk for mortality during pregnancy, a 25-50% risk for death. Management of labor and delivery is particularly problematic. These women are at greatest risk when there is diminished venous return and right ventricular filling which is associated with most maternal deaths. Similar mortality rates are seen in aortic coarctation with valve involvement and Marfan syndrome with aortic involvement. The baby is not at risk for pulmonary hypoplasia unless there is very preterm rupture of the fetal membranes.

A 24-year-old Rh-negative G2P1 woman is found at 10 weeks gestation to have anti-D antibodies. You follow her closely during this pregnancy and order serial ultrasound examinations. Which of the following fetal ultrasound findings would be most explained by the presence of Rh disease? A. Meconium B. Fetal bladder obstruction C. Oligohydramnios D. Pericardial effusion E. Placenta previa

D. Fetal hydrops is easily diagnosed on ultrasound. It develops in the presence of decreased hepatic protein production. It is defined as a collection of fluid in two or more body cavities, such as ascites, pericardial and/or pleural fluid and scalp edema. On occasion, when extramedullary hematopoiesis is extensive, there will be evidence of hepatosplenomegaly. Placentomegaly (placental edema) and polyhydramnios are also seen on ultrasound. Meconium, fetal bladder obstruction, oligohydramnios and placenta previa do not fit the clinical scenario.

A 29-year-old G1P0 woman at 28 weeks gestation who is the wife of basketball player is diagnosed with gestational diabetes. Her BMI is 23.8. Her mother had a delivery complicated by shoulder dystocia and she is concerned about her own risk. Which of the following is her biggest risk factor for shoulder dystocia? A. Family history B. Tall husband C. Maternal weight D. Gestational diabetes E. Parity

D. Fetal macrosomia, maternal obesity, diabetes mellitus, postterm pregnancy, a prior delivery complicated by a shoulder dystocia, and a prolonged second stage of labor are all associated with an increased incidence of shoulder dystocia. Although a family history can be indicative of large babies which might place her at additional risk, her gestational diabetes represents her largest risk factor.

A 24-year-old G2P1 woman at 30 weeks gestation is sensitized to the D antigen. She is Rh-negative and received RhoGAM after her first delivery one year ago. Which of the following statements best explains these findings? A. The patient initiated her prenatal care late during the present pregnancy B. The patient was sensitized during the previous pregnancy by receiving the RhoGAM C. Current pregnancy is too close to the first pregnancy D. The amount of fetal maternal hemorrhage was more than previously estimated E. The cause is most likely idiopathic in this case

D. On rare occasion, an Rh-negative woman will subsequently be sensitized, despite prophylaxis. The protection afforded by a standard RhoGAM administration is dose-dependent. One dose will prevent Rh sensitization to an exposure of as much as 30 cc of Rh-positive red blood cells. With greater exposure, there is only partial protection and Rh sensitization may occur as a result of failure to diagnose massive transplacental hemorrhage. Alternatively, an Rh-negative woman may be sensitized in the latter part of pregnancy or soon after delivery before the post-delivery prophylaxis dose is given. Inadvertent maternal transfusion of Rh-positive blood may result in Rh sensitization to the D or another red blood cell antigen. Patients may become sensitized if they do not receive RhoGAM following an episode of antenatal bleeding or after an invasive procedure, such as amniocentesis or chorionic villus sampling. In addition, RhoGAM only confers protection against the D antigen. Therefore, despite administration of RhoGAM to Rh-negative patients, they may still become sensitized to other red blood cell antigens. Pregnancy spacing does not affect the presence of the antibody.

A 34-year-old G1 woman at eight weeks gestation presents for prenatal care. She is healthy and takes no medications. Family history reveals type 2 diabetes in her parents and brothers. She is 5 feet 2 inches tall and weighs 220 pounds (BMI 40.2 kg/m2). Which of the following is the best recommendation to screen her for gestational diabetes? A. Screen at 24-28 weeks with a 50-g oral glucose challenge test B. Screen at 16-20 weeks with a 50-g oral glucose challenge test C. Screen at 12 weeks with a 50-g oral glucose challenge test D. Screen now with a 50-g oral glucose challenge test E. Begin an oral hypoglycemic agent now

D. Screening should be performed between 24 and 28 weeks in those women not known to have glucose intolerance earlier in pregnancy. This evaluation can be done in two steps: a 50-g oral glucose challenge test is followed by a diagnostic 100-g oral glucose tolerance test (OGTT) if initial results exceed a predetermined plasma glucose concentration. Patients at low risk are not routinely screened. For those patients of average risk screening is performed at 24-28 weeks while those at high risk (severe obesity and strong family history) screening should be done as soon as feasible.

A 34-year-old G4P3 woman at 36 weeks with a twin gestation presents in labor. She has three prior normal spontaneous vaginal deliveries at term, with the largest infant weighing 3400 grams. Twin A is breech with an estimated fetal weight of 2800 gm and twin B is vertex, with an estimated fetal weight of 3200 gm. Which of the following is an appropriate delivery option for this patient? A. Total breech extraction of twin A, vaginal delivery of twin B B. External cephalic version for twin A, vaginal delivery twin of B C. Operative vaginal delivery for twin A and vaginal delivery for twin B D. Cesarean delivery E. Vaginal delivery for twin A and Cesarean delivery for twin B

D. The optimal mode of delivery for twins in which the first twin is in the breech presentation is by Cesarean section. Similar to singletons, if the first twin is breech problems can occur including head entrapment and umbilical cord prolapse. When the presenting twin is vertex and twin B is not vertex, controversy exists as to the optimal mode of delivery. A small randomized study comparing Cesarean delivery with vaginal delivery for vertex-non-vertex twins failed to show an advantage for Cesarean delivery, but did not have statistical power to address rare neonatal morbidities. Some authors have advocated external cephalic version for management of the second twin; however, observational studies have not shown any advantage of this approach compared to total breech extraction.

A 20-year-old G1P0 woman has vaginal spotting and mild cramping for the last three days. Her last normal menstrual period was approximately seven weeks ago. She had a positive home pregnancy test. Vital signs are: blood pressure 120/72; pulse 64; respirations 18; temperature 98.6°F (37°C). On pelvic exam, she has scant old blood in the vagina, with a normal appearing cervix and no discharge. On bimanual exam, her uterus is nontender and small, and there are no adnexal masses palpable. Quantitative Beta-hCG 48 hours ago was 750 mIU/ml. Today, current Beta-hCG 760 mIU/ml; progesterone 3.2 ng/ml; hematocrit 37%. Transvaginal ultrasound shows a fluid collection in the uterus with a yolk sac but no fetal pole. A 3x3 cm cyst is seen on the left ovary. There is no free fluid in the pelvis. Which of the following is the most appropriate next step in the management of this patient? A. Exploratory laparoscopy B. Treat with methotrexate C. Treat with mifepristone D. Expectant management E. Repeat ultrasound in one week

D. The pregnancy is abnormal based on the abnormal Beta-hCG levels and the progesterone level. In a normal pregnancy, the level should rise by at least 50% every 48 hours until the pregnancy is 42 days old (after that time, the rise in level may not follow the projection curve). A progesterone level of <5 ng/ml suggests an abnormal or extrauterine pregnancy. In this instance, the pregnancy is intrauterine because of the presence of a yolk sac. Expectant management is appropriate and may avoid the risks of surgery. Other options include misoprostol, manual vacuum aspiration, or dilation and curettage. Laparoscopy and methotrexate are not indicated as this is a confirmed intrauterine pregnancy. Mifepristone is a progestin receptor antagonist and can be used as emergency contraception to prevent ovulation and blocks the action of progesterone which is needed to maintain pregnancy. In the US, Mifepristone is also used with misoprostol for pregnancy termination.

A 33-year-old G2P1 woman presents at 12 weeks gestation for routine prenatal visit. She has had an uncomplicated prenatal course. Doppler fetal heart tones are not heard and the ultrasound today shows a crown rump length of 8 mm with no cardiac activity and a retroverted uterus. What is the next step in the management of this patient? A. Check a serum progesterone level B. Obtain serial Beta-HCG levels every two days C. Repeat the ultrasound in seven days D. Medical induction with misoprostol E. Medical induction with methotrexate

D. This patient has a missed abortion and should be offered uterine evacuation. Ultrasound criteria for a missed abortion are a CRL of > 7 mm with no cardiac activity. Medical induction using misoprostol has been shown to be efficacious and associated with less complications when compared to surgical evacuation. Checking a serum progesterone and following serial Beta-hCG may be indicated in confirming a viable pregnancy. Methotrexate is used in the treatment of selected ectopic pregnancies and can be used to induce medical terminations of pregnancies if the LMP was < six weeks ago.

An 18-year-old G1 woman at 32 weeks gestation presents with severe abdominal pain and a small amount of bleeding. She has received routine prenatal care, smokes one pack of cigarettes per day and admits to using crack cocaine. On exam, her blood pressure is 140/80; pulse 100; and she is afebrile. Her uterus is tense and very tender. Pelvic ultrasound reveals a fundal placenta, cephalic presentation of the fetus and no other abnormalities. Cervical examination reveals blood coming through the os and is one centimeter dilated. Fetal heart tones have a baseline of 160s, with a category III tracing. Which of the following is the most likely diagnosis? A. Placenta previa B. Premature rupture of the membranes C. Preterm labor D. Placental abruption E. Chorioamnionitis

D. This patient has a placental abruption. Common presenting signs of an abruption include abdominal pain, bleeding, uterine hypertonus and fetal distress. Risk factors include smoking, cocaine use, chronic hypertension, trauma, prolonged premature rupture of membranes, and history of prior abruption. Treatment would involve an emergent Cesarean delivery with appropriate resuscitation, including intravenous fluids and blood products as needed. A placenta previa is an abnormal location of the placenta. Chorioamnionitis is an infection that typically occurs following prolonged rupture of membranes, and is frequently accompanied by a maternal fever. Premature rupture of membranes presents with loss of fluid not active bleeding.

An 18-year-old G1P0 woman presents at 32 weeks for a routine visit. She complains of intense itching for the past two weeks and cannot stop scratching her arms, legs, and soles of her feet. She has tried over the counter lotions and antihistamines with no relief. She also states that her family noticed she is slightly yellow. Her vital signs are normal and there are scattered excoriations over her arms and legs. Which of the following is the best treatment in the management of this patient? A. Aggressive hydration B. Antivirals such as Acyclovir C. Antihistamines D. Ursodeoxycholic acid E. Steroids

D. This patient has pruritus gravidarum, a common pregnancy-related skin condition that is a mild variant of intrahepatic cholestasis of pregnancy. There is retention of bile salt, and as serum levels increase they are deposited in the dermis. This, in turn, causes pruritus. The skin lesions are secondary to scratching and excoriation. Antihistamines and topical emollients may provide some relief and should be used initially. Ursodeoxycholic acid relieves pruritus and lowers serum enzyme levels. Another agent reported to relieve the itching is the opioid antagonist naltrexon. Hydroxychloroquine is used to treat lupus and is not indicated in this patient.

A 27-year-old G2P1 woman presents to the emergency department with increasing lower abdominal pain, nausea, scant bleeding, and fever. She is two days postop from a suction dilatation and curettage for an incomplete abortion. Vital signs: blood pressure 120/80, pulse 104, respiratory rate 20, and temperature 100.4°F (38.0°C). Physical examination reveals rebound tenderness and abdominal guarding, uterus soft and slightly tender. Which of the following is most likely in this patient? A. Normal recovery symptoms B. Retained products of conception C. Hematometra D. Perforated uterus E. Cervical laceration

D. This patient has the classic signs and symptoms of a uterine perforation. Hematometra can develop after an abortion, but the patient would complain of cyclic midline abdominal cramping pain. Retained products of conception would cause profuse vaginal bleeding and if not removed may lead to a septic abortion. Complications that may occur secondary to suction dilatation and curettage include anesthesia risk, bowel and bladder injury, cervical lacerations, and uterine perforations. If the patient had a cervical laceration she would have more vaginal bleeding than scant.

A 32-year-old G5P3 woman presents with left-sided abdominal pain. Her last normal menstrual period was eight weeks ago. She began having pain early this morning and it has increased to a severity of 8/10. She denies nausea or vomiting or vaginal bleeding. Her gynecological history is notable for a right-sided ectopic pregnancy four years ago. At that time, she had a right salpingectomy and a left tubal ligation. On physical examination: blood pressure is 90/54; pulse 108; respirations 22; and temperature 98.6°F (37.0°C). On abdominal examination, she has rebound and guarding in all quadrants, and on pelvic exam, her uterus is very tender and there is left adnexal fullness. Urine pregnancy test is positive. A transvaginal ultrasound shows a thickened endometrium, left pelvic mass with a gestational sac and fetal pole, and a large amount of free fluid in the pelvis. Her hematocrit is 26%. What would be the next best step in the management? A. Admit for observation B. Repeat Beta-hCG level in 48 hours C. Treat with methotrexate D. Perform a laparoscopy E. Perform a dilation and curettage

D. This scenario is consistent with the patient having a ruptured ectopic pregnancy. Signs of hypovolemia (tachycardia, hypotension) with peritoneal signs (rebound, guarding and severe abdominal tenderness) and a positive pregnancy test lead to the diagnosis of ruptured ectopic pregnancy. Conservative management, with observation and repeating the Beta-hCG level in 48 hours is not indicated since a diagnosis is clear and delaying surgery can potentially be dangerous to the patient. Dilation and curettage would only be considered after laparoscopy, if needed.

A 27-year-old G1P0 woman at 32 weeks gestation presents complaining of cough, fever, chest pain, and dyspnea. Vital signs are pulse 108; temperature 100.5° F (38.0° C); respiratory rate 22 per minute. Physical examination reveals right lower lobe bronchial breath sounds. Which of the following tests would be most appropriate for making a diagnosis for this patient? A. Blood cultures B. Sputum culture C. Lower extremity dopplers D. Chest x-ray E. Pulmonary function tests

D. This woman presents with classic symptoms and findings for pneumonia. The typical symptoms include cough, dyspnea, sputum production, and pleuritic chest pain. Mild upper respiratory symptoms and malaise usually precede these symptoms, and mild leukocytosis is usually present. Chest radiography is essential for diagnosis, although radiographic appearance does not accurately predict the etiology of the pneumonia. Pulmonary function tests, blood and sputum cultures, serological testing, cold agglutinin identification, and tests for bacterial antigens are not recommended in uncomplicated pneumonia.

A 39-year-old G4P1 woman at 36 weeks gestation presents to labor and delivery. Upon initial evaluation, no fetal heart tones were noted on Doptone. Ultrasound confirms fetal demise. Problems during the pregnancy include diagnosis of an open neural tube defect, estimated fetal weight >90th percentile, polyhydramnios and a nonreactive NST (non-stress test) the week prior to admission. What is the most likely etiology of this fetal demise? A. Uncontrolled hypertension B. In-utero viral infection C. Antiphospholipid antibody syndrome (APAS) D. Uncontrolled diabetes E. Untreated maternal hypothyroidism

D. Uncontrolled diabetes during organogenesis is associated with a high rate of birth defects. The most common sites affected are the spine and the heart of the fetus, although all birth defects are increased. Fetuses in utero exposed to high levels of glucose transplacentally have increased growth and polyuria resulting in an increase in the amniotic fluid. While some viral infections are also associated with placentomegaly and polyhydramnios, the fetus will have normal or decreased growth depending on the timing of the infection. Severe hypertension and active APAS is often associated with oligohydramnios and intrauterine growth restriction. The risk of miscarriage is increased if hypothyroidism goes untreated.

A 32-year-old G1P0 woman at 10 weeks gestation presents to your office after an ultrasound evaluation has revealed a diamniotic, dichorionic twin gestation. She is very concerned about the risk for preterm delivery. Which intervention would you recommend as a possible means to reduce the risk of a preterm, low-birthweight infant? A. Bed rest B. Cervical cerclage C. Tocolytics starting at 24 weeks D. Home uterine monitoring E. Early, good weight gain

E. Although prematurity has been recognized as a major cause of morbidity and mortality among twin gestations, interventions for prevention of prematurity have, in general, been unsuccessful. Studies show that an adequate weight gain in the first 20 to 24 weeks of pregnancy is especially important for women carrying multiples and may help to reduce the risk of having preterm and low-birth weight babies. These pregnancies tend to be shorter than singleton pregnancies, and studies suggest that a good early weight gain aids in development of the placenta, possibly improving its ability to pass along nutrients to the babies. Bed rest, long prescribed by obstetricians for the prevention of preterm birth, has never been shown to be efficacious, and may be associated with thromboembolic complications. An observational study of prophylactic cerclage for twin gestations failed to show any benefit. Tocolytic drugs for prevention of preterm labor in asymptomatic women with twin gestations have not been shown to be effective. Home uterine activity monitoring is another intervention that has been shown to be ineffective.

A 24-year-old G2P1 woman is diagnosed with Rh hemolytic disease at 24 weeks gestation. Measurement of which of the following in the amniotic fluid is best indicative of the severity of the disease? A. Hemoglobin B. Iron C. Anti-D antibody titer D. Ferritin E. Bilirubin

E. In the presence of a severely erythroblastotic fetus, the amniotic fluid is stained yellow. The yellow pigment is bilirubin, which can be quantified most accurately by spectrophotometric measurements of the optical density between 420 and 460nm, the wavelength absorbed by bilirubin. The deviation from linearity of the optical density reading at 450nm is due to the presence of heme pigment, an indicator of severe hemolysis. Amniotic fluid ferritin, an acute-phase reactant, is associated with spontaneous preterm delivery and not with Rh isoimmunization.

A 24-year-old Rh-negative G2P1 woman at 18 weeks gestation is positive for anti-D antibodies. In discussing the risks of Rh sensitization with her, you tell her that her fetus may be at increased risk of significant perinatal disease including fetal anemia. Which of the following non-invasive tests can detect severe fetal anemia? A. Umbilical artery systolic-diastolic ratio B. Biophysical profile C. Amniotic fluid index D. Umbilical artery blood flow E. Middle cerebral artery peak systolic velocity

E. Noninvasive diagnosis of fetal anemia can be performed with Doppler ultrasonography. The use of middle cerebral artery peak systolic velocity in the management of fetuses at risk for anemia because of red cell alloimmunization has emerged as the best test for the noninvasive diagnosis of fetal anemia. All the other listed tests are for assessment of fetal well-being and non-specific to detect fetal anemia. Amniocentesis and cordocentesis have been used for many years to diagnose fetal anemia due to red cell alloimmunization. These techniques, however, are invasive and many complications are associated with their use.

A 34-year-old G1P0 woman presents with vaginal spotting. On physical exam: blood pressure 120/70; pulse 82; respirations 20; and temperature 98.6°F (37.0°C). An ultrasound confirms a non-viable intrauterine pregnancy. She is otherwise healthy. Her partner accompanies her and is supportive. The patient wishes to avoid any unnecessary medical interventions and asks whether she can safely let nature take its course. What is the best next step in the management of this patient? A. Immediate dilation and suction curettage B. Dilation and suction curettage in one week C. Immediate treatment with misoprostol D. Treatment with misoprostol in one week E. Expectant management

E. Patients experiencing early pregnancy loss can safely consider several different treatments, including expectant management, medical treatment to assist with expulsion of the pregnancy or surgical evacuation. Provided the patient is hemodynamically stable and reliable for follow-up, expectant management is appropriate therapy. At the gestational age described, expectant management portends no increase in risk of either hemorrhage or infection compared with surgical or medical evacuation. Regardless of method chosen, the patient's blood type should be checked and rhogam administered as indicated.

A 17-year-old G1P0 female at 39 weeks gestation presents with increased swelling in her face and hands over the last two days. Her blood pressure is 155/99. She has 2 plus pitting edema of the lower extremities. A 24-hour urine collection shows 440 mg of protein. What is the next best step in the management of this patient? A. Fluid restriction B. Magnesium sulfate C. Furosemide D. Hydralazine E. Delivery

E. Regardless of disease severity, the only definitive therapy for preeclampsia is delivery of the fetus and placenta. This solution can occasionally be delayed in the setting of stable disease (mild or severe) when it occurs at an extremely early gestational age. Fluid management must be monitored closely in this person. Magnesium sulfate is the mainstay of therapy during labor and for 24 hours postpartum to lower the seizure threshold in women with severe disease. Low-dose aspirin may have some benefit in decreasing the risk of preeclampsia in a subset of high-risk patients. Hydralazine is often the antihypertensive agent of choice for controlling elevated blood pressures in the acute setting.

A 27-year-old G2P1 woman at 18 weeks gestation presents to the emergency department complaining of fever, nausea, vomiting, and mid-abdominal pain for the last 24 hours. For the last 12 hours, she has had no appetite. She has been healthy, but reports that her three-year-old son has had diarrhea for two days. Physical examination reveals a blood pressure of 100/60; pulse 88; respiratory rate 18; and temperature 102.0°F (38.9°C). Abdominal examination reveals decreased bowel sounds and tenderness more pronounced on the right than the left. Which of the following is the next best step in the management of this patient? A. Complete blood count B. X-Ray of the abdomen C. Helical CT scan D. MRI E. Graded compression ultrasound

E. Suspected appendicitis is one of the most common indications for surgical abdominal exploration during pregnancy. The diagnosis is made based on clinical findings and graded compression ultrasonography that is sensitive and specific especially before 35 weeks gestation. This noninvasive procedure should be considered first in working up suspected acute appendicitis. Selective imaging of the appendix using helical computed tomography may be a safe and potentially reliable tool to accurately identify appendiceal changes in appendicitis, except that radiation exposure using this test is higher than graded compression ultrasonography. An MRI would not expose the patient to radiation but is not the best diagnostic study for appendicitis. A plain abdominal radiograph can be used to identify air fluid levels or free air but offers little diagnostic value for appendicitis. The diagnosis of appendicitis is more difficult to make in pregnancy because anorexia, nausea, and vomiting that accompany normal pregnancy are also common symptoms of appendicitis. In addition, the enlarged uterus shifts the appendix upward and outward toward the flank, so that pain and tenderness may not be located in the right lower quadrant. Appendicitis is easily confused with preterm labor, pyelonephritis, renal colic, placental abruption, or degeneration of a uterine myoma. Peritonitis and appendiceal rupture are more common during pregnancy.

A 23-year-old G1P0 woman at 40 weeks gestation presents to labor and delivery with contractions. At 10:00 am, her cervical exam is 2 centimeters dilated, 70% effaced and the vertex at 0 station. Clinical pelvimetry reveals an adequate pelvis and membranes are intact. The fetus is in a cephalic presentation and EFW is 3500 gms. Contractions are occurring every 3-4 minutes, based on the external monitor. Her labor slowly progresses and, at 1:00 pm, the patient has spontaneous rupture of membranes. Fetal surveillance remains Category I. Her cervical exam is 5 centimeters dilated, 100% effaced, and 0 station. At 4:00 pm, the patient's cervical exam is unchanged. Contractions are occurring every 5-6 minutes. Which of the following is the most appropriate next step in the management of this patient? A. Perform a biophysical profile B. Have the patient ambulate C. Perform a Cesarean delivery D. Continue fetal surveillance and reexamine the patient in two hours E. Begin oxytocin augmentation

E. The patient has an arrest of dilatation in the active phase of labor. She is only having contractions every 5-6 minutes, so it is reasonable to start oxytocin to increase the frequency and strength of this patient's contractions. If the patient does not have cervical change once she is having more frequent contractions on oxytocin, it would be reasonable to place an IUPC (intrauterine pressure catheter) to assess the strength of the contractions. It is not yet necessary to perform a Cesarean delivery. Further observation and having the patient ambulate do not facilitate delivery. A biophysical profile is not indicated in this situation.

19-year-old G1 woman presents at 28 weeks gestation for prenatal care. Her past medical history is unremarkable except for a splenectomy following a motor vehicle accident four years ago. Prenatal labs today show a hemoglobin of 12 g/dL; blood type O positive; Rh negative with antibody screen positive for Lewis (titer 1:16). What is the next best step in the management of this pregnancy? A. Check father of the baby's antibody status B. Biophysical profile C. Serial amniocentesis D. Percutaneous umbilical blood sampling E. Reassurance

E. The patient should be reassured that the fetus is not at risk even though the antibody titer is 1:16. Lewis antibodies are IgM antibodies and do not cross the placenta, therefore are not associated with isosensitization or hemolytic disease of the fetus. The father of the baby does not need to be tested nor does this unaffected fetus need a biophysical profile. The other tests listed above are invasive and used to monitor fetuses at risk for anemia, hydrops and fetal death.

A 34-year-old G2P1 woman at 18 weeks gestation presents with a newly discovered lump in her left breast. Fine needle aspiration reveals adenocarcinoma. Which of the following is the least likely recommended therapy for breast cancer during pregnancy? A. Wide local excision biopsy B. Modified radical mastectomy C. Total mastectomy and node dissection D. Chemotherapy E. Radiotherapy

E. There is no doubt that breast cancer is more aggressive in younger women. Whether it is more aggressive during pregnancy in young women is debatable. Slight delays (1 to 2 months) in clinical assessment, diagnostic procedures, and treatment of pregnant women with breast tumors are common. Approximately 30 percent of pregnant women with breast cancer have stage I disease, 30 percent have stage II, and 40 percent stages III or IV. Many clinical reports maintain that when breast cancer is diagnosed during pregnancy, the regional lymph nodes are more likely to contain microscopic metastases. Surgical treatment may be definitive for breast carcinoma during pregnancy and in the absence of metastatic disease a wide excisional biopsy, modified radical mastectomy, or total mastectomy with axillary node staging can be performed. Non-pregnant women receive adjunctive radiotherapy with breast-conserving surgery. However, this is not recommended during pregnancy due to sizeable abdominal scatter placing the fetus at significant risk for excessive radiation.

A 31-year-old G1 woman presents at term in active labor. After four hours her cervix is still 5 cm dilated and the fetal station is zero. The fetal heart rate tracing is Category III. The patient's contractions have increased in intensity and are occurring every 2 to 3 minutes. Which of the following is the most appropriate next step in the management of this patient? A. Ambulation B. Ultrasound for estimated fetal weight C. Continued monitoring of labor D. Augmentation with oxytocin E. Cesarean delivery

E. This patient needs a Cesarean delivery since she is remote from delivery with a Category III fetal heart rate tracing. Continued monitoring of labor would be appropriate if the patient was making adequate progress and there was a Category I fetal heart rate tracing. Augmentation of labor would be indicated if there was a reassuring fetal heart rate tracing and the patient was not making adequate progress of labor. An ultrasound at this stage of labor is inaccurate and ambulation in the presence of a non-reassuring fetal heart tracing is contraindicated.

A 27-year-old G2P1 woman at 36 weeks gestation is admitted for preeclampsia with severe features. Her blood pressure is 200/105. Diastolic blood pressures during her pregnancy have ranged from 50-60 mmHg. She has received two doses of IV hydralazine to lower her blood pressure. What diastolic blood pressure should you aim for in this patient? A. 50-55 mmHg B. 60-65 mmHg C. 70-75 mmHg D. 80-85 mmHg E. 90-95 mmHg

E. Treatment with an antihypertensive is indicated for blood pressures persistently greater than 160 systolic and 105 diastolic. First-line agents include hydralazine (a direct vasodilator) 5 mg IV followed by 5-10 mg doses IV at 20-minute intervals (maximum dose = 40 mg); or labetalol (combined alpha & beta-adrenergic antagonist) 10-20 mg IV followed by 20 mg, then 40 mg, then 80 mg IV every 10 minutes (maximum dose = 220 mg). The goal is not a normal blood pressure, but to reduce the diastolic blood pressure into a safe range of 90-100 mmHg to prevent maternal stroke or abruption, without compromising uterine perfusion.


Set pelajaran terkait

Null Hypothesis & Significance Testing

View Set

Revature Pre-placement Evaluation

View Set

Chapter 09 Skeletal System: Articulations

View Set

CULTURE: CENTRAL, SOUTH, EAST, AND SOUTHEAST ASIA

View Set

bio 2 test 2 Final Exam Practice

View Set

Chapter 16: Agency Relationships

View Set